You are on page 1of 369
2009 UPDATES IN THERAPEUTICS THE PHARMACOTHERAPY PREPARATORY COURSE AMERICAN COLLEGE OF CLINICAL PHARMACY UPDATES IN THERAPEUTICS: THE PHARMACOTHERAPY PREPARATORY COURSE 2009 EDITION VOLUME II accep Director of Professional Development: Nancy M. Perrin, M.A., CAE. Project Manager, Education: Emma F. Webb, M.A. Medical Editor: Kimma Sheldon, Ph.D. Desktop Publisher/Graphic Designer: Jen DeYoe, BA For order information or questions, contact: ‘American College of Clinical Pharmacy 13000 W. 87th St. Parkway Lenexa, KS 66215-4530 Phone: (913) 492-3311 Fax: (913) 492-0088 scep@acep.com hitp/vorw.acep.com Copyright© 2009 by the American College of Clinical Pharmacy. All rights reserved. This book is protected by copyright. No ‘part of this publication may be reproduced, stored ina retrieval system, or transmitted, in any form or by any means, electronic ‘or mechanical, including photocopy, without prior written permission of the American College of Clinical Pharmacy. Printed in the United States of America. To properly cite this book: Author(s). Chapter name. In: Bailey, T, Bamette D, Bressler L, et al. Updates in Therapeutics: The Pharmacotherapy Preparatory Course, 2009 ed. Lenexa, KS: American College of Clinical Pharmacy, year:pages. Note: The authors and publisher ofthe Pharmacotherapy Preparatory Course recognize that the development ofthis ‘material offers many opportunities for error. Despite our very best efforts, some errors may persist into print. Drug dosage schedules are, we believe, accurate and in accordance with current standards. Readers are advised, however, to check other published sources to be certain that recommended dosages and contraindications are in agreement with those listed in this ‘book. This is especially important in the cases of new, infrequently used, or highly toxic drugs. ISBN-13: 978-1-932658-65-1 ISBN-10: 1-932658-65-3, Continuing Pharmacy Education: ‘The American College of Clinical Pharmacy is accredited by the Accreditation Council for Pharmacy Education as a provider of continuing pharmacy education. The Universal Activity Numbers are: Pharmacotherapy Preparatory Course for home study, 2009 Edition: Pediatrics, Geriatrics, and Oncology Supportive Care, 217-000-09-013-HOI-P, 2.0 contact hours; Biostatistics: A Refresher and Clinical ‘rials: Fundamentals of Design and Interpretation, 217-000-09-014-H01-P, 3.0 contact hours; Infectious Diseases, HIV/Infectious Diseases and Pharmacokinetics: A Refresher, 217-000-09-015-HO1-P, 3.0 contact hours; ‘Neurology and General Psychiatry, 217-000-09-016-HOI-P, 2.0 contact hours; Acute Care Cardiology and Critical Care, 217-000-09-017-H01-P, 3.0 contact hours; Nephrology, Endocrine and Metabolic Disorders and Fluids, Electrolytes and Nutrition, 217-000-09-018-HO1-P, 3.0 contact hours; Ambulatory Care; and Men's and Women’s Health., 217-000-09-019-HO1-P, 3.0 contact hours; Gastrointestinal Disorders and Outpatient Cardiology, 217-000-09-020-HO1-P, 2.5 contact hours. ‘To eam continuing pharmacy education credit forthe home study version of the 2009 Pharmacotherapy Preparatory Course, you must succesfully complete and submit the Web-based posttest associated with each program within the couse by not later than October 31, 2010. Statements of continuing pharmacy education credit will be available at www. acep,com within 4 weeks of submission of the Web-based post tes, "The American College of Clinical Pharmacy (ACCP) has compiled the materials in this course book for pharmacists to use in preparing for the Board of Pharmaceutical Specialties (BPS) Pharmacotherapy Specialty Certification Examination. ‘There is no intent or assurance that all of the knowledge on the examination will be covered in the ACCP process. Although ACCP does use the BPS Content Outline in creating the material for this course, ACCP does not know the specific content of any particular BPS examination, BPS guidelines prohibit any overlap of individuals writing the examination and developing preparatory materials PROGRAM GOALS AND TARGET AUDIENCE Updates in Therapeutics: The Pharmacotherapy Preparatory Course is designed to help pharmacists who are preparing for the Board of Pharmaceutical Specialties certification examination in Pharmacotherapy as well as, those seeking a general review and refresher on disease states and therapeutics. The program goals are: 1, To present a high-quality, up-to-date overview of disease states and therapeutics; 2. To provide a framework to help attendees prepare for the specialty certification examination in pharmacotherapy; and 3. To offer participants an effective learning experience using a case-based approach with a strong focus on the thought processes needed to solve patient care problems in each therapeutic area. FACULTY Teresa M. Bailey, Pharm.D., BCPS Associate Professor Ferris State University Kalamazoo, Michigan Debra J. Barnette, Pharm.D., BCPS Ambulatory Care Coordinator University of North Carolina Chapel Hill, North Carolina Linda R. Bressler, Pharm.D., BCOP Clinical Associate Professor Director of Regulatory Affairs (Cancer and Leukemia Group B) University of Illinois Chicago, Iinois G. Robert DeYoung, Pharm.D., BCPS Clinical Pharmacist, Ambulatory Care Advantage Health Physicians and St. Mary’s Health Care Grand Rapids, Michigan Edward F. Foote, Pharm.D., FCCP, BCPS Associate Professor Wilkes University Wilkes Barre, Pennsylvania Brian Hemstreet, Pharm.D., BCPS Associate Professor University of Colorado at Denver and Health Sciences Center Aurora, Colorado ‘Tudy M. R. Hodgman, Pharm.D. Critical Care Pharmacist Northwest Community Healthcare Arlington Heights, Illinois William A. Kehoe, Pharm.D., FCCP, BCPS Professor of Clinical Pharmacy and Psychology Chairman, Department of Pharmacy Practice University of the Pacific Stockton, California Judith Kristeller, Pharm.D., BCPS Associate Professor Wilkes University Wilkes Barre, Pennsylvania Kirsten H. Obler, Pharm.D., BCPS Clinical Assistant Professor University of Illinois Chicago, Ilinois Norma J. Owens, Pharm.D., FCCP, BCPS Professor and Chair Pharmacy Department University of Rhode Island Providence, Rhode Island Robert Lee Page, I, Pharm.D., FCCP, FAHA, BCPS Associate Professor of Clinical Pharmacy and Physical Medicine Schools of Pharmacy and Medicine University of Colorado Aurora, Colorado Jo E. Rodgers Pharm.D., BCPS Clinical Associate Professor Division of Pharmacotherapy and Experimental Therapeutics School of Pharmacy University of North Carolina Chapel Hill, North Carolina Melody Ryan, Pharm.D., BCPS Associate Professor University of Kentucky Lexington, Kentucky Curtis L. Smith, Pharm.D., BCPS Professor Ferris State University Lansing, Michigan Consultancies: Speaker's Burea Other: Nothing to Disclose: FACULTY DISCLOSURES Brian Hemstreet (Axcan Pharma, Conexus Health); Robert Page (ArcaBio Pharma, Astra Zeneca); Jo Rodgers (Arca Discovery, Scios, Inc., Astellas, The ‘Medicines Company). Edward Foote (American Regent); Brian Hemstreet (Axcan Pharma, AstraZeneca); Jo Rodgers (Scios, Inc.) ‘Norma Owens (Steere House Nursing Home and Rehabilitation Center); Melody Ryan (Teva). Teresa Bailey; Debra Barnette; Linda Bressler; G. Robert DeYoung; Tudy Hodgman; William Kehoe; Judith Kristeller; Kirsten Ohler; Curtis Smith. ACKNOWLEDGMENTS ‘Teresa M. Bailey, Pharm.D, BCPS [Asocate Profesor Collegeof Pharmacy Fes State University Kalapazoo, Michigan Debra J. Barnett, Pharm.D. BCPS Ambulatory Care Coordinator Univesity of No Carli (Chapel Hil, North Carona Lisa Anne Booth, Pharm.D. BCPS CGlinal Research Phamacist. Drug Information Coordinator Pharmacy Adminisration Columbis Regional Healthcare System Columbus, Goren Linda R, Bressler, Pharm.D, BCOP CGlneal Asolate Professor Departneat of Pharmacy Practice, College of Pharmacy Director of Regltry Affairs (Cancer and Lesko Gop B Univesity of ings ‘Chicago, Minis Gretchen M. Brophy, PharmD. BCPS, FCCP, FCCM ‘Associate Presa of Pharmacy and Netronurgey Schoo of Pham (htc! Care Phanacit [Netrostene Intensive Cae Unit Medial College of Vigna Virginia Commoaweat University Ricknond,Vigiia Kathleen M, Bungny, Pharm.D. FCCP Reseach Scien’ ‘The Heath Instte ‘New England Medical Center Clinical Pharmacie ‘New England Medical Center ‘Asis Profesor of Peyetsty Tufts Univesity Sebo! af Medicine Bost, Massachsets 6. Robert DeYoung, PharmD. BCPS award F Foote, Pharm.D, FCCP, BCPS, [Asointe Professor Wilkes University ‘Wiles Bae, Pensyvania Brian A. Hemstret, Pharm.D, BCPS ‘AssoratePrfesor Deparment of Clinical Pharma, Sol of Pharmacy ‘Unversity of Colorado at Denver and Heath Scicnces Centr Denver, Clordo ‘Trudy MR. Hodgman, Phara.D, BCPS Ceca Cae Pharmacist "Northwest Community Healbeare Artington Heights, inois ‘Willam A. Kehoe, Pharm.D. MLA FCCP, BCPS Profesor of Clinical Pharmacy and Paycholony (Caiman, Deparment of Pharmacy Practice ‘TJ Long Sol of Pharmacy ad Heath Sciences Univers ofthe Pace ‘Stockton, California odin Krister, Pharm.D, BCPS ‘Associate Profesor ‘Wakes Universi, ‘Wiles Bare, Pensyvania Harold J. Manley Pharm.D, BCPS. [Asrisie Profesor ‘Department of Pharmacy Practice ‘Albany Collegeof Pharmacy “Albany, New York Kiesen H Ohler, PharmD., BCPS (Clinia Asien Professor Universi of ios Chicag, Minos ‘Norma J. Owens, Pharm.D, FCCP, BCPS Professor and Cai Phammacy Depart University of Rode stand Providence, Rhode sland ‘Robert Lee Page, Il Pharm.D, FCCF, FAH, BCPS Associate Profesor of Clinical Pharmacy and Physical Medicine ‘School f Pharmacy and Medicine Universi of Colenco| ‘Aor, Colorado Jo E Rodgers Pharm.D, BCPS ‘Clinical Atoclate Profesor Division of Purmacotieapy ond Experimental Therapeutics ‘School of Pharmacy University of Noth Carotina (Chapel Hil, Now Cra ‘Melody Ryan, Pharm.D, BCPS ‘Associate Profesor Department of Pharmacy Practice and Science CClige of Phamacy ‘University of Ketucky Lexingon, Kentucky Gordan 8 Sacks, PharmD., BCNSP, FCCP (Cink Prfeso od Cie Phanacy Pace Division, School of Pharmacy University of Wisconsin ~ Madson Madison, Miscou Lise A. Sanchez, Pharm, Preient PE Applications Highlands Ranch, Colorado Curtis, Smith, Pharm.D, BCPS Profesor Fes State Universi Lansing, Michigan “Anne . Spencer, Pharm.D. BCPS ‘Asocnte Profesor of Pharmacy and Clinical Sciences Collegeof Pharmacy Medial University of South Cactina CCaceston, Sout Carona (Ceresa'T. Ward, Pharm.D, BCPS, (Clinseal Coordinator Emory Crford Long Hospital ‘Anta, Georgi Erle. Witbrodt, Pharm.D, FCCP, BCPS, ‘Medic! Edventon and Resch Lion Medi! Afi ‘Tap Pharmaceutical Products ne ‘Veerbees, New Jersey TABLE OF CONTENTS NEPHROLOGY. Acute Renal Failure; Drug- Induced Renal Damage; Renal ‘Replacement Therapy; Complications of Chronic Kidney Disease; Dosage Adjustments in Renal Insufficiency ENDOCRINE AND METABOLIC DISORDERS. Diabetes Mellitus; Diabetes Ketoacidosis; Nonketotic Hyperghve Hyperosmolar State; Diabetes Complications; Hyperlipidemia in Diabetes; Diabetic Nephropathy; Diabetic Retinopathy; Foot Care; Immunization; Thyroid Disease; Adrenal Disorders FLUIDS, ELECTROLYTES, AND NUTRITION.. Regulatory Issues; Fluid Requirements; Sodium Homeostasis; Potassium Homeostasis; Magnesium Homeostasis; Phosphorus Homeostasis; Calcium Homeostasis; Acid Base Disorders; Nutrition 2-67 AMBULATORY CARE. Asthma; Chronic Obstructive Pulmonary Disease; Anticoagulation; Disorders of Lipid Metabolism; Adult Immunizations MEN'S AND WOMEN’S HEALTH. Osteoporosis; Hormone Replacement Therapy: «and Lactation; Complications in Pregnaney; Contraception; Sexually Transmitted Diseases; Prostatic Infections; Male Sexual Dysfunction GASTROINTESTINAL DISORDERS. Gastroesophageal Reflux Disease; Peptic Ulcer: Gastrointestinal Bleeding; Inflammatory Bowel Disease; Complications of Alcoholic Liver Disease; Viral Hepatitis OUTPATIENT CARDIOLOGY... Heart Failure; Atrial Fibrillation; Hypertension; Disease; Chronic Stable Angina seosene 2-227 ADDITIONAL RESOURCES TYPES OF ECONOMIC AND HUMANISTIC OUTCOMES ASSESSMENTS. From the ACCP publication, “Pharmacoeconomics and Outcomes" POLICY, PRACTICE, AND REGULATORY ISSUES. HIPAA, IRB, and Informed Consent; Prescription Drug Approval Process; Investigational Drug Service; JCAHO, ORYX, NCQA, ‘and HEDIS THE BOARD OF PHARMACEUTICAL SPECIALTIES “2009 CANDIDATE'S GUIDE’. Printed courtesy ofthe Board of Pharmaceutical Specialties NEPHROLOGY EDWARD F. FOOTE, PHARM.D., BCPS, FCCP WILKES UNIVERSITY WILKES-BARRE, PENNSYLVANIA NEPHROLOGY EDWARD F. FOOTE, PHARM.D., BCPS, FCCP WILKES UNIVERSITY WILKES-BARRE, PENNSYLVANIA © 2009 American College of Clinical Pharmacy 21 ‘Nephrology Learning Objectives: 1, Categorize acute kidney injury (AKI) as prerenal, intrinsic, or postrenal, based on patient history, physical examination, and laboratory values. 2. List risk factors for AKI and formulate strategies. to decrease risk of AKI in specific patient populations. 3. Develop a care plan to manage AKI. 4, Identify medications and medication classes associated with acute and chronic kidney damage, 5. Discuss factors that determine the efficiency of dialysis of drugs. For specific agents, calculate the amount of drug removed by dialysis. 6. Identify the stage of chronic kidney disease (CKD) based on patient history, physical examination, ‘and laboratory values. 7. List risk factors for the progression of CKD and. formulate strategies to slow the progression of CKD. 8, Describe the common complications of CKD. 9. Develop a care plan to manage the common complications observed in patients with CKD (eg., anemia, secondary hyperthyroidism). Self-Assessment Questions: Answers to these questions may be found at the end of. this chapter. 1. AM. is a 75-year-old man who presents to your institution complaining of abdominal pain’ and inability to urinate during the past 2 days. Physical cxamination reveals a slightly distended abdomen, His CHEM-7 profile shows sodium (Ne) 135 mEaq/L; chloride (Cl) 108 mEq/L; potassium (K) 45 mEq/L; CO, 29 mEa/L; blood urea nitrogen (BUN) 17 mg/dL; serum creatinine 2.0 mg/dL; and glucose 48 mg/dL. He has no known drug allergies. His weight is 92.5 kg, and his height is 611”. Which one of the following is best to include in the initial management of this patient? A. Administer furosemide 40 mg intravenously x B. Insert Foley catheter to check for residual urine C. Administer fluid bolus (1000 mL of normal saline solution), followed by furosemide 40 ‘mg intravenous push. D. Administer Humalog insulin 3 units ‘subcutaneously. ED. is a 44-year-old man admitted with gram- negative bacteremia. He receives 4 days of parenteral aminoglycoside therapy and develops acute tubular necrosis. Antibiotic therapy is adjusted based on culture and sensitivity results. ‘Which one of the following is the urinalysis most likely to show? A. BUN/serum creatinine ratio greater than 20:1; urine Na less than 10 mOsml/L; fractional excretion of sodium (FENa) less than 1%; specific gravity more than 1.018; hyaline casts B, BUN/serum creatinine ratio greater than 20:1; urine Na more than 20 mOsmV/L; FENa ‘more than 3%; specific gravity 1.010; no casts visible. C. BUN/serum creatinine ratio 10-15:1; urine ‘Na more than 40 mOsmU/L; FENa more than 1%; specific gravity less than 1.015; muddy cass, D. BUN/serum creatinine ratio 10-15:1; urine Tess than 10 mOsmi/L; FENa less than 1%; specific gravity more’ than 1.018; muddy casts, W.C. is a patient with CKD stage 4 (estimated creatinine clearance [CrCl] of 25 mL/minute) ‘The patient has a diagnosis of gram-positive bacteremia, which is susceptible only to drug X. ‘There are no published reports on how to adjust the dose of drug X in patients with diminished kkidney function. Review of the drug X package insert shows that it has significant renal climination, with 40% excreted unchanged in the urine. The usual dose for drug X is 600 me/day intravenously and is provided as 100 mg/mL. in a 6-mL vial. How many milliliters of drug X should be given to W.C.? AL 4m. B. 43m. C 36mL. D. SSmL. © 2009 American College of Clinical Pharmacy 22 ‘Nephrology 4. DZ. is a 45-year-old patient with a long history of cancer. He has long-standing malnutrition and is well below his ideal body weight. He is to be dosed on carboplatin, in which an accurate estimate of kidney function is critical. Which one of the following is the best method for assessing kkidney function in this patient? A. Cockeroft-Gault. B. Modification of diet in renal disease (MDRD). C. 24-hour collection of urine. D. Iothalamate study. 5. WC. is a $0-year-old man with type 2 diabetes ‘mellitus who presents to your clinic this afternoon for evaluation of his kidney function. Dipstick results of spot urine are +3 for albumin. Which ‘one of the following is the best test to perform next to evaluate proteinuria in this patient? A. Perform quantitative test such as random urine for albumin-to-Cr ratio, B, Reschedule an appointment to obtain a first- ‘morning albumin dipstick. . Repeat albumin dipstick test in 6 months. D. Noother laboratory tests are required; initiate enalapril 20 mg orally 2 times/day. 6. QRS. is a 60-year-old (72 kg) patient in the intensive care unit. He suffered @ myocardial infartion about I week ago with secondary heart failure. He now has pneumonia. He has ‘been hypotensive for the past 5 days. Before his admission 1 week ago, he had a serum creatinine of 1.0 mg/dL. His medical history is significant for diabetes mellitus and hypertension. His urine ‘output has been steadily declining for the past 3 days, despite adequate hydration. He made only 700 ml. of urine during the past 24 hours. His medications since surgery include intravenous dobutamine, nitroglycerin, and cefazolin Yesterday, his BUN/serum creatinine was 32 and 3.1 mpldL; today, itis 41 and 3.9 mg/dL. His urine osmolality is 290 mOsm/kg. His urine Na is 40 mEq/L, and there ae tubular celular casts in his urine. Which one of the following is the most likely renal diagnosis? A. Prerenal azotemia B. Acute tubular necrosis. C. Acute interstitial nephritis. D. Hemodynamicifunctional-mediated AKL 7. You are evaluating study comparing epoetin and darbepoetin on their efficacy on mean hemoglobin concentrations. Both drugs are started at the recommended dose, and the ‘hemoglobin concentration is checked at 4 weeks. There are 50 patients in each group. The mean hemoglobin in the epoetin group is 12.1 g/L, and in the darbepoetin group, it is 12.2 g/L. Which one ofthe following statistical tests is best for this comparison? ‘A paired ttest. ‘An independent (unpaired) t-test. ‘An analysis of variance. ‘A chi-squared test. pop> © 2009 American College of Clinical Pharmacy 23 Nephrology Patient Cases 1, H.D. isa 48-year-old African American man (70 kg) admitted to the intensive care unit after an acute myocardial infarction. His medical history is significant for type 2 diabetes mellitus diagnosed 1 year ago and for hypertension and tobacco use. His current medications are metformin 500 mg orally 2timesiday; amlodipine 5 mglday orally; nicotine patch 14 mg/day applied each morning; and naproxen $00 mgiday orally. Before admission, his kidney function was normal (serum creatinine = 1.0. mg/dL); however, during the past 24 hours, his kidney function has declined (BUN 20 mg/L, serum creatinine 2.1, mg/dL). His urine demonstrates muddy casts. The patients anuric. His current blood pressure is 120/80 mm Hig. Which one ofthe following isthe best assessment of H.D: kidney function? A. 26.2 mL/minute (CrCl using Cockerof-Gaul) B. 44‘mL/minute/173 m: (glomerular filtration rate (GFR) using abbreviated MDRD), €. 23.1 ml-/mimute/0 kg (CrCl using the Brater equation). D. Assume CrCl less than 10 mL minute 2. Which one ofthe following represents the most likely cause of kidney dysfunetion inthis patient? Prerenal Intrinsic. Postrenal Functional. pomp 3. Which of the following medications is best to discontinue at this time? ‘A. Amlodipine. B. Naproxen, C. Metformin and amlodipine. D._ Metformin and naproxen. I. ACUTE KIDNEY INJURY (ACUTE RENAL FAILURE) ‘A. Definitions and Background i. Acute kidney injury is defined as an acute decrease in kidney function. 1. Definitions vary. A commonly used definition is an increase in a serum creatinine of 0.5 mg/dL or greater or a decrease of 25% or greater in the GFR of patients with previous normal kidney function OR an increase of 1 mg/dL or greater in serum creatinine in patients with CKD. Also defined based on urine output (less than 0.5 mL/kg/hour for at Teast 6 hours) 2. Common complications include fluid overload as well as acid-base and electrolyte abnormalities. 3. Urine output classification a. Anuric: less than 50/24 hours—associated with worse outcomes b. Oligurie: 50-500 mL/24 hours ¢. Nonoliguric: more than 500 mL/24 hours—associated with better patient outcomes. Easier to manage because of fewer problems with volume overload fi, Community-acquired AKT 1. Low incidence (0.02%) in otherwise healthy patients. As high as 13% in patients with cKD 2. Usually has a very high survival rate (70%-95%) 3. Single insult to the kidney, often drug induced 4, Often reversible iii, Hospital-acquired AKT 1. Has a moderate incidence (2%-5%) and moderate survival rate (30%-50%) 2. Single or multifocal insults to the kidney 3. Can still be reversible © 2009 American College of Clinical Pharmacy 24 Nephrology iv. Intensive care unit-acquired AKI. Has a high incidence (6%-23%) and a low survival rate (10%-30%) . v. Estimating GFR in AKI 1. Difficult because commonly used equations (Cockeroft-Gault and MDRD) are not appropriate (need stable serum creatinine) 2. Equations by Brater and Jellife are probably more accurate than Cockcroft-Gault but have not been rigorously tested. . Risk Factors Associated with AKI i. Volume depletion—vomiting, diarrhea, poor fluid intake, fever, diuretic use (effective volume depletion; congestive heart failure [CHF]) Preexisting CKD Use of nephrotoxic agents/medications (intravenous radiographic contrast, aminoglycosides, amphotericin, nonsteroidal antiinflammatory drugs [NSAIDs] and cyclooxygenase-2 [COX- 2] inhibitors, angiotensin-converting enzyme inhibitors [ACEls] and angiotensin II receptor blockers [ARBs], cyclosporine, and tacrolimus) iv. Obstruction of the urinary tract Classifications of AKI (Table 1) Prerenal azotemia 1. Initially, the kidney is undamaged, 2. Characterized by hypoperfusion to the kidney i. Systemic hypoperfusion: hemorrhage, volume depletion, drugs, CHF ii, Isolated kidney hypoperfusion: renal artery stenosis, emboli 3. Urinalysis will initially be normal (no sediment) but concentrated. 4, Physical examination: hypotension, volume depletion Functional AKT 1. Kidney is undamaged. Often “lumped” with prerenal in classification 2. Caused by reduced glomerular hydrostatic pressure 3. Generally medication related (cyclosporine, ACEIs and ARBs, and NSAIDs). Seen in patients with underlying effective blood flow (patients with CHF, patients with liver disease, and elderly patients) who cannot compensate for alterations in afferentefferent tone. Concentrated urine 4, Small increases in serum creatinine (less than 1 mg/dL) after initiation of ACEVARB are acceptable. . Intrinsic azotemia 1, Kidney is damaged. Damage can be linked to structure involved: smail blood vessels, glomeruli, renal tubules, and interstitium. 2. Most common cause is acute tubular necrosis. Others include acute interstitial nephritis, vasculitis, and acute glomerulonephritis. 3. Urinalysis will reflect damage. Urine generally not concentrated 4, Physical examination: normotensive, euvolemic, or hypervolemic; check for signs of allergic reactions or embolic phenomenon 5. History: identifiable insult, drug use, infections iv. Postrenal 1. Kidney is initially undamaged. Bladder outlet obstruction is the most common cause of posirenal AKI. Lower urinary tract obstruction may be due to calculi. Ureteric obstructions may be due to clots or intraluminal obstructions. Extrarenal compression can also cause postrenal disease. Increased intraluminal pressure upstream of the obstruction will result in damage if obstruction is not relieved. © 2009 American College of Clinical Pharmacy 25 ‘Nephrology 2. Urinalysis may be nonspecific. 3. Physical examination: distended bladder, enlarged prostate 4, History: trauma, benign prostatic hypertrophy, cancers ‘Table 1. Classifications of Acute Kidney Injury Prerenal and Functional Tatinsie Postrenal istory/presentation — | Volume depletion ischemic failure [Kidney stones |Renal artery stenosis, {Nephrotoxins (¢.g.,contrast) BPH CHE Vasculitis [Cancers Hypercalcemia |Glomerulonephritis, NSAID/ACEI use [Cyclosporin Physical examination |Hypotension ‘AIN: rash, fever Distended bladder Dehydrat Enlarged prostate Petechia if thrombotic Ascites Serum BUNSGr ratio [> 20:1 isa is: [Urine concentrated? [Yes No No Low urine Na (< 20 mEq/L) |Urine Na>40 mEq/L [Urine Na> 40 mEq/L Low FENa (<1) FENa>2 FENa>2 High UOsm. Low UOsm [Low UOsm Urine sediment [Normal [Muddy-brown granular casts; | Variable, may be normal tubular epithelial casts Urinary WBC. [Negative [a-4+ Variable (Urinary RBC [Negative 2-4 + Proteinuria [Negative [Positive Negative ACEI = angotensia-converting enzyme inhibitor, AIN= eute interstitial nephritis, BPH ~ benign prostatic hypertrophy; BUN = blod urea ‘itogen; CHF = congestive bear falure; FENa~ factional excretion of sodium; GFR (glomerular ltation rate), NSAID = nonsteroidal an- {inflammatory rug; SCr = serum creatinine; RBC ~ ed blood cell (coun); WBC D. Prevention of AKI "preferred. Patient education Avoid: dopamine, fenoldopam Drug therapies to decrease incidence of contrast-induced nephropathy—see drug nephrotoxicity section vi. Tight glycemic control Avoid nephrotoxic drugs when possible. Ensure adequate hydration (2 L/day). If intravenous, 0.9% sodium chloride (NaCl) is, diuretics E, Treatment and Management of Established AKT i, Prerenal azotemia: correct primary hemodynamics Normal saline if volume depleted 2. Pressure management if needed 1 3. ii, Postrenal azotemi Blood products if needed. iii, Intrinsic: no specific therapy universally effective Eliminate the causative hemodynamic abnormality or toxin. 1. 2. 3. Avoid additional insults. Prevent and treat complications. a. Fluid management ite blood esl cout), lieve obstruction. Consult urology and/or radiology. i, Maintain kidney perfusion and production of urine. Consider fluid bolus. © 2009 American College of Clinical Pharmacy 26 Nephrology ii, Diuretic therapy: Consider for patients who are oliguric and euvolemic o hypervolemic. a. Loop diuretics: i, Give intravenously. (a) Furosemide intermittent therapy: 40-80 mg intravenously every 6-8 hours. May increase bolus dose up to 400 meg if no response (b) Furosemide continuous infusion: 40-80 mg intravenous bolus and then 10-20 mg/hour intravenously ii, Continuous intravenous therapy and/or combination with thiazide- like diuretics may decrease resistance. b. Dopamine therapy. Renal dose dopamine (1-2 meg/kg/minute) causes renal and mesenteric vasodilatation and can increase urinary output. Not proved to improve resolution of kidney failure. Not recommended b. Acidosis i, Restrict dietary protein (less than 0.5 g/kg/day of high-quality protein). ii, Sodium bicarbonate to maintain HCO, more than 15 mEq/L and arterial pH ‘more than 7.2 iii, Dialysis ¢. Electrolyte and nutrition abnormalities—supportive. See fluid, electrolyte, and nutrition section, vi, Renal replacement therapy—see indications below. Can give hemodialysis or peritoneal dialysis Patient Cases 4. EB. is a 67-year-old man referred to cardiology for intermittent chest pain. The patient has a history significant for CKD, type 2 diabetes mellitus, and hypertension. His home medications include enalapril, hydrochlorothiazide, and pioglitazone. Laboratory values include serum creatinine 1.8 mg/dL, glucose 189 mgidL, hemoglobin 12 mg/dL, and hematocrit 36%. His physical examination is normal. The plan is to undergo elective cardiac catheterization. Which one of the following choices best characterizes the risk factors for developing contrast-induced nephropathy inthis patient? A. CKD and diabetes, B. CKD, diabetes, anemia, and enalapril C. Diabetes, anemia, and age. D. Diabetes 5. The physician plans to start 0.9% normal saline for hydration. Which one of the following best represents the ‘optimal time to start the intravenous infusion in relation to the timing of the procedure? A. 24hours prior. B. 12 hours prior. C. 2hours prior. D. Immediately prior. 6. In addition to intravenous fluid, which one of the following therapies is best to use in E.P. to decrease his likelihood of developing contrast-induced nephropathy’ Fenoldopam, Acetyleysteine, “Ascorbic acid, Hemofiltration, pop © 2009 American College of Clinical Pharmacy 27 ‘Nephrology Il, DRUG-INDUCED KIDNEY DAMAGE A. Introduction: Drugs are responsible for kidney damage through multiple mechanisms, Evaluate potential drug-induced nephropathy based on the period of ingestion, patient risk factors, and propensity of the suspected agent to cause kidney damage. i, Definitions vary. Increase in serum creatinine of 0.5 mg/dL or more or decrease of 25% or ‘more in GFR in patients with previous normal kidney function OR an increase of | mg/dL or ‘more in serum creatinine in patients with CKD fi, Risk factors 1. History of CKD 2. Increased age iii, Epidemiology 1, 7% of all drug toxicities 2, 18%-27% of AKI in hospitals 3. 1%-5% of NSAID users in community 4, Most implicated medications: aminoglycosides, NSAIDs, ACEIs, contrast dye, amphotericin iv. Kidney at risk of toxicity because: 1. High exposure to toxin: Kidney receives 20%-25% cardiac output. 2. Autoregulation and specialized blood flow through glomerulus 3. Intrakidney drug metabolism 4, Tubular transport processes 5. Concentration of solutes (e., toxins) in tubules 6. High energy requirements of tubule epithelial cells, 7. Urine acidification Pseudo- 50% and! or ferritin > $00 Initial test dose | Yes; 25-mg one-time test dose [No INo (CRD = cvonisWdney Geass HD = bemodalyi IVP = IV push; IVPB =1V piggyback, NSS = nomal saline solutions TSAT= wanserin B, Renal Osteodystrophy and Secondary Hyperparathyroidism i, Pathophysiology: Calcium and phosphorous homeostasis is complex; it involves the interplay ‘of hormones affecting the bone, gastrointestinal tract, kidneys, and PTH. Process may begin as early as GFR 60 mL/minute. The most important driving force behind the process is hyperphosphatemia! Nephron loss: decreased production of 1,25 dihydroxyvitamin D3 and phosphate retention. Increased phosphorous concentrations cause 1) an inhibition of vitamin D activation, reducing absorption of calcium in the gut; 2) a decrease in ionized (free) calcium concentrations; and 3) direct stimulation of PTH secretion. Elevated PTH concentrations cause decreased reabsorption of phosphorus and increased reabsorption of calcium in the proximal tube. This adaptive mechanism is lost as the GFR falls below 30 mL/ ‘minute. Important: Calcium is not well absorbed through the gut at this point, and calcium concentrations are maintained by increased bone resorption through elevated PTH. Unabated calcium loss from the bone results in renal osteodystrophy. ii, Prevalence 1. Major cause of morbidity and mortality in patients undergoing dialysis 2. Very common iii, Signs and symptoms 1. Insidious onset: Patients may complain of fatigue and musculoskeletal and gastrointestinal pain; calcification may be visible on radiography; bone pain and fractures ‘can occur if progression is left untreated. © 2009 American College of Clinical Pharmacy 2-22 Nephrology 2. Laboratory abnormalities a. Phosphorus b. Corrected calcium ¢. Intact parathyroid hormone iv, Treatment 1, Goals of therapy—Table 3 Table3. [KDOQI Guidelines for Calcium, Phosphorus, CaxPO4 Product, and Parathyroid Hormone in CKD Stages 3-5 CKD Stage 3 CKD Stage 4 CKD Stage 5 (Calcium (mg/dL) ‘Normal ‘Normal 849.5 Phosphorus (mg/dL) 27-46 27-46, 3555 (Ca x POS product <55 <35 <35 PTH (pgimL) 35-10 70-110 150-300 “Use comected caleium = serum calcium > (08 » [4.0 patient albumin), (CKD = chronic kidney disease; KDOQI = Kidney Disease Outcomes Quality Inittve; PTH = parathyroid hormone, 2. Nondrug therapy a, Dietary phosphorus restriction 800-1200 mg/day in stage 3 CKD or higher b. Dialysis removes various amounts of phosphorus depending on treatment modalities, but, by itself, is insufficient to maintain Phos balances in most patients. ¢. Parathyroidectomy. Reserved for patients with unresponsive hyperparathyroidism 3. Drug therapy Phosphate binders: Take with meals to bind phosphorus in the gut; products from different groups may be used together for additive effect. i, Aluminum-containing phosphate binders (aluminum hydroxide, aluminum carbonate, sucralfate). Effectively lowers phosphorus concentrations. In general, avoid. Not used as often because of aluminum toxicity (adynamic bone disease, encephalopathy, and erythropoietin resistance) ii, Caleium-containing phosphate binders (calcium carbonate and calcium acetate) a. Widely used binder. Considered initial binder of choice. Carbonate is relatively inexpensive. b. Carbonate is also used to treat hypocalcemia, which sometimes occurs in patients with CKD and can also decrease metabolic acidosis. ¢. Calcium acetate. 667-mg capsule contains 167 mg of elemental calcium. Better binder than carbonate, so less calcium given Use may be limited by development of hypercalcemia. €. Total elemental calcium per day = 2000 mg/day (1500-mg binder; 500-mg, diet) iii, Sevelamer: a nonabsorbable phosphate binder a, Effectively binds phosphorus b. Consider if calcium-phosphorus factor is greater than 55 mg'/dL?. ¢. Decreases low-density lipoprotein cholesterol and increases high-density lipoprotein cholesterol 4. Hypocalcemia may occur if sevelamer is sole phosphate binder. Metabolic acidosis may worsen with sevelamer HCI. ©. Available as sevelamer HCl (Renagel) and sevelamer carbonate (Renvela) © 2009 American College of Clinical Pharmacy 2.23 Nephrology iv, Lanthanum carbonate: a. As effective as aluminum in phosphate-binding capability. Also considered first line b. Tasteless, chewable wafer c. Consider if calcium x phosphorus product is more than 55 mg?/dL. ‘Vitamin D analogs: Suppress PTH synthesis and reduce PTH concentrations; therapy is limited by resultant hypercalcemia, hyperphosphatemia, and elevated calcium- Phosphorus product. Products include Calcitrol, doxercalciferol, and paricalcitol. Calcitriol is the pharmacologically active form of 1,2 hydroxyvitamin D3; U.S. Food and Drug Administration (FDA) label approved for the ‘management of hypocalcemia and the prevention and treatment of secondary hyperparathyroidism a. Oral and parenteral formulations 'b, Does not require hepatic or renal activation c. Low-dose daily oral therapy reduces hypocalcemia but does not reduce PTH concentrations significantly. 4. High incidence of hypercalcemia limiting PTH suppression ©. Dose adjustment at 4-week intervals ii, Paricalcitol: vitamin D analog; FDA label approved for the treatment and prevention of secondary hyperparathyroidism a. Parenteral and oral formulation b. Does not require hepatic or renal activation ¢. Lower incidence of hypercalcemia (decreased mobilization of caleium from the bone and decreased absorption of calcium from the gut) Doxercalciferol: vitamin D analog; FDA label approved for the treatment and prevention of secondary hyperparathyroidism a. Parenteral and oral formulation b. Prodrug, requires hepatic activation; may have more physiologic levels c. Lower incidence of hypercalcemia (decreased mobilization of calcium from the bone and decreased absorption of calcium from the gut) Cinacalcet HCI: A calcimimetic that attaches to the calcium receptor on the parathyroid gland and increases the sensitivity of receptors to serum calcium concentrations, thus reducing PTH. Especially useful in patients with high calciumy ‘phosphate concentrations and high PTH concentrations when vitamin D analogs cannot be used i, The initial dose is 30 mg, irrespective of patient PTH concentration. ii, Monitor serum calcium every 1-2 weeks (risk of hypocalcemia is about 5%); do not start therapy if serum calcium is less than 8.4 mg/L. iii, Can be used in patients irrespective of phosphate binder or vitamin D analog use iv. Caution in patients with seizure disorder (hypocalcemia may exacerbate) v. Adverse effects are nausea (30%) and diarthea (20%). vi. Cytochrome P450 2D6 metabolism: Dose reductions in drugs with narrow therapeutic indexes may be required (flecainide, tricyclic antidepressants, and thioridazine). vii, Ketoconazole increases cinacalcet concentrations up to twofold. © 2009 American College of Clinical Pharmacy 2:24 Nephrology Patient Case 13. PP. isa 40-year-old dialysis patient with a history of grand mal seizures. He takes phenytoin 300 mg/day. His albumin concentration is 3.0 g/L. His total phenytoin concentration is 5.0 mg/dL. Which one ofthe following is the best interpretation of the phenytoin concentrations? ‘The concentration is subtherapeutic, and a dose increase is warranted, ‘The concentration is therapeutic, and no dosage adjustment is needed, ‘The concentration is toxic, and a dose reduction is needed. The level is not interpretable. voR> ‘VI. DOSAGE ADJUSTMENTS IN KIDNEY DISEASE A. Dosages of many drugs will require adjustment to prevent toxicity in patients with CKD; adjustment strategies will vary, depending on whether the patient is receiving renal replacement ‘therapy and, if so, the type of renal replacement therapy. B. Pharmacokinetic Principles Can Guide Therapy Adjustments. i. Absorption 1, Oral absorption can be decreased. a. Nausea and vomiting . Increased gastric pH (uremia) c. Edema 4. Physical binding of drugs to phosphate binders ii, Distribution 1. Changes in concentrations of highly protein-bound and highly water-soluble drugs occur as extracellular fluid status changes. 2. Acidic and neutral protein-bound drugs are displaced by toxin buildup. Other mechanisms include conformational changes of plasma protein-binding site. Phenytoin is a classic example. a. Hypoalbuminemia correction Concentration adjusted = concentration measured/{(0.2 * measured albumin) + 0.1] b. Renal failure adjustment Concentration adjusted = concentration measured/{(0.1 x measured albumin) + 0.1] cc. Patients will have lower total concentrations despite having adequate free concentrations. 4. Dosage adjustment of phenytoin not needed, just a different approach to evaluating ‘concentrations iii, Metabolism, Variable changes can occur with uremia, Metabolites can accumulate. iv, Exeretion ~ Decreased C. Pharmacodynamic Changes Can Also Occur. Example: Patients with CKD can be more sensitive to benzodiazepines. D. General Recommendations: i, Patient history and clinical data ii, Estimate CrCl (Jeliffe or Brater in AKI; Cockeroft-Gault in stable kidney function). iii, Identify medications that require modification (Table 4), ‘© 2009 American College of Clinical Pharmacy 2.25 ‘Nephrology ‘Table 4, Dose Adjustments in Decreased Kidney Function ‘Agent Dose Adjustment (Antibiotics [Almost all antibiotics will require dosage adjustment (exceptions: cloxacillin, clindamycin, linezolid, metronidazole, and macrolides) ‘Cardiac medications ‘Atenolol, ACEIs, digoxin, nadolol, sotalol; avoid potassium-sparing diuretics if (CrCl < 30 mL/minute [Lipid-lowering therap (Clofibrate, fenofibrate, statins [Narcotics (Codeine, avoid meperidine; other agents may also accumulate [Antipsychotic/antiepileptic |Chloral hydrate, gabapentin, lithium, paroxetine, primidone, topiramate, lagents. trazodone, vigabatrin Hypoglycemic agents [Acarbose, chlorpropamide, glyburide, glipizide, insulins, and metformin [Antiretrovirals Individualize therapy: monitor CD, counts, viral load, and adverse effects (agents requiring dose adjustment: lamivudine, adefovir, didanosine, stavudine, tenofovir, zalcitabine, and zidovudine) Miscellaneous ‘Allopurinol, colchicine, H,-receptor antagonists, diclofenac, Ketorolac, and terbutaline [ACEI = angotensin-convering enzyme inhibitors; CC ~ creatinine clearance, iv. Calculate drug doses individualized for the patient. 1. Published data 2. Rowland-Tozer estimate a. Q=1-[Fe(l-KF)] b. Q=kinetic parameter or drug dose adjustment factor c. Fe= fraction of drug excreted unchanged in the urine 4. KF = ratio of patients’ CrCl to normal (120 mL/minute) Monitor patient (e.g., kidney function; clinical parameters) and drug concentration (if applicable). Revise regimen as appropriate. E, Drug Dosing in Hemodialysis i iii, Dosing changes in hemodialysis patients may be necessary because of accumulation due to kidney failure AND/OR because the procedure may remove the drug from the circulation. Drug-related factors affecting drug removal during dialysis: 1. Molecular weight—With high-flux membranes, larger molecules (such as vancomycin) can be removed. 2. Water soluble—nonsoluble drugs not likely removed 3. Protein binding—Because albumin cannot pass through membranes, protein-bound drugs cannot either. 4, Volume of distribution—Drugs with a small V, (less than 1 L/kg) available in central circulation for removal. Large V,s cannot be removed (digoxin and tricyclic antidepressants), even if the protein binding is very low. Procedure-related factors affecting drug removal 1, Type of dialyzer—high flux widely used now 2. Blood flow rate, Increased rates will increase delivery and maintain gradient across membrane. 3. Duration of dialysis session 4, Dialysate flow rate, High rates of flow wil ‘across membranes. increase removal by maintaining the gradient © 2009 American College of Clinical Pharmacy 2-26 Nephrology REFERENCES National Kidney Foundation’s Kidney Disease Outcome Quality Initiative 1, Go to www-kidney.org/professionals/ ‘Acute Kidney Injury 1. Hilton R, Acute kidney injury [review]. BMJ 2006;333:786-90. 2. Ympa YP, Sakr Y, Reinhart K, Vincent JL. Has ‘mortality from acute kidney injury decreased? A systematic review of the literature, Am J Med 2005;118:827-32, 3. Dager W, Spencer A. Acute renal failure. In: DiPiro JT, Talbert RL, Yee GC, et al., eds Pharmacotherapy: A Pathophysiologc Approach, ‘rn ed. New York: McGraw-Hill, 2008:723-43, 4. StamatakisMK. Acute renal failure. In: Chisholm- Bums MA, Wells BG, Schwinghammer TL, etal., eds. Pharmacotherapy: Principles and Practice. New York: McGraw-Hill, 2007:chapter 22. Drug-Induced Kidney Damage 1, Nolin TD, Himmelfarb J. Drug-induced kidney disease, In: DiPiro JT, Talbert RL, Yee GC ail, eds. Pharmacotherapy: A Pathophysiologic Approach, 7th ed. New York: McGraw-Hill, 2008:795-810. 2. Schweiger MJ, Chambers CE, Davidson CI, et al. Prevention of contrast induced nephropathy: recommendations for the high risk patient undergoing cardiovascular procedures. Catheter Cardiovasc Interv 2007;69:135-40. Chronic Kidney Disease 1. Joy MS, Ksirsagar A, Franceshini N. Chronic Kiney disease: progression-modifying therapies, In: DiPito JT, Teldest RL, Yee eds. Pharmacotherapy: A Pathophysiologic Approach, 7th ed. New York: MeGraw-Hill, 2008:745-64 2. Parmar MS. Chronic renal disease. BMJ 2002;325:85-90. 3. Hudson JQ. Chronic kidney disease: management of complications. In: DiPiro JT, Talbert cal, eds. Pharmacotherapy: A Pathophiysiologic Approach, 7th ed. New York: McGraw-Hill, 2008:765-92. 4. Schonder KS. Chronic and end-stage renal disease, In: Chisholm-Bums MA, Wells BG, Schwinghammer TL, etal, eds. Pharmacotherapy: Principles and Practice. New York: McGraw-Hill, 2007 chapter 23. Renal Replacement Therapy 1. Foote EF, Manly HJ. Hemodialysis and peritoneal dialysis In: DiPiro JT, Talbert RL, Yee GC, al, eds. Pharmacotherapy: A Pathophysiologic Approach, 7th ed. New York: McGraw-Hill, 2008:793-94, 2. Piraino P, Bailie GR, Bemardini J, etal. Peritoneal dialysis-related infections recommendations: 2008 update. Perit Dial Int 2005;25:107-31. ‘Complications of CKD 1. Pharmacotherapy specialists should be aware of the National Kidney Foundation Kidney Disease Outcome Quality initiative Web site. Available at _htpyiwwwkidney.org/professionals/kdogi ‘guidetines.cfin, Accessed February 21, 2007. 2 Hudson JQ. Chronic kidney disease: management of complications. In: DiPiro JT, Talbert RL, Yee GC, et al, eds. Pharmacotherapy: A Pathophysiologic Approach, 7th ed. New York: McGraw-Hill, 2008:765-92. 3. Schonder KS. Chronic and end-stage renal disease. In: Chisholm-Bums MA, Wells BG, ‘SchwinghammerL, eta.,eds. Pharmacotherapy: Principles and Practice. New York: McGraw-Hill, 2007:chapter 23. Drug Therapy Adjustment in CKD Kappel J, Calissi P. Nephrology: 3. Safe drug prescribing for patients with renal insufficiency. Can Med Assoc J 2002;166:473-7. 2. Matzke GR, Frye RE. Drug individualization for patients with renal insufficiency. In: DiPiro JT, Telbert RL, Yoo GC. ct al, eds. Pharmacotherapy: A Pathophysiologic Approach, 7th ed. New York: McGraw-Hill, 2008:833-44, therapy © 2009 American College of Clinical Pharmacy 2.27 Nephrology ANSWERS AND EXPLANATIONS TO PATIENT CASES, 1. Answer: D Estimating CrCl in a patient with unstable kidney function is difficult. The Jeliffe or Brater equation has. been recommended as preferable to other equations. In this case, the patient is anuric; hence, a CrCl (GER) of 10 mL/minute or less (Answer D) should be assumed. Answer A (Cockerof-Gault is inappropriate because Cockeroft-Gault should only be used with stable kidney function. The use of MDRD (Answer 'B) in unstable kidney function is also inappropriate. ‘Although Answer C, the Brater equation, may be used, it still overestimates kidney function in this patient because the patient is anuric. 2. Answer: B ‘This patient very likely has acute tubular necrosis, which isa type of intrinsic renal failure (Answer B). ‘The rapid rise in serum creatinine, the BUNICr ratio ‘of about 10, and the muddy casts all point to acute tubular necrosis. There is no evidence of prerenal causes (hypotension, volume depletion) (Answer A). Naproxen is associied with functional AKI, but the ‘urine in these patients is bland without casts. Answer C js incorrect because there is no evidence of obstruction in this patient 3. Answer: D One of the strategies in the management of AKI is to remove potentially nephrotoxic drugs. It is common, to see the following orders for patients in AKI: no ACEls, ARBs, NSAIDs, or intravenous contrast. It is also important to remove (or reduce the dose of) agents that are cleared renally. Metformin accumulates. in decreased kidney function with an increased risk of lactic acidosis and should be temporarily discontinued. at this time. Because the patient is normotensive, there is no compelling need to discontinue the amlodipine. Review drug therapy of patients with AKI, removing. "unnecessary agents and correcting doses for decreased. kidney function, 4. Answer: B ‘The most important risk factors for contrast-induced nephropathy include the presence of CKD in a patient with diabetes. In men, a hematocrit less than 39% is also a risk factor. Enalapril may worsen the effects of contrast. The cutoff for age to be a risk factor is 75 vars. 5. Answer: B It is recommended to start intravenous normal saline 6-12 hours before the procedure. Answers C and D would likely not allow enough time for adequate hydration and diuresis. Answer A is reasonable, but there is no compelling reason to start that early, and. this may prolong the hospital stay. Some recent data ‘with sodium bicarbonate intravenously show benefit ‘with a more narrow start time (2 hours). 6. Answer: B Much data have published on the use of oral acetyleysteine in the prevention of contrast-induced nephropathy. Although many of the studies were observational, the low risk of the product has made it the standard of eare in this situation. Fenoldopam (Answer A) should not be used based on the results of the CONTRAST trial. Thereare somedata ithascorbic acid (Answer C), but they are limited. Hemofiltration (Answer D) has also been studied but is not generally recommended because of the questionable benefits and the real risk of complications. TeAmswer: A He is currently at stage 2 CKD (GFR 60-89 mL/ ‘minute/1.73 m*), which can be calculated by use of the MDRD formulae or Cockcroft-Gault. The five stages correlate from mild kidney damage (stage 1) to kidney failure (stage 5). 8. Answer: C Based on his long history of diabetes and overt proteinuria, this patient likely has diabetic nephropathy. Progression will be accelerated by smoking, poor diabetes control, and poor blood pressure control. In patients with diabetes, a target hemoglobin (Hg)A,,. ‘of less than 7 g/dL. is associated with a decrease it the rate of disease progression. Blood pressure control less than 130/80 mim Hg in patients also decreases the progression of kidney disease. The standard of care in patients with diabetic nephropathy are ACEIs (or ARBs). A nondihydropyridine (Answer B) might be ‘initiated in patients who cannot tolerate ACE or ARB therapy. Dihydropyridine therapy (Answer A) is not recommended in diabetic nephropathy because of conflicting literature on its efficacy. An increase in atenolol (Answer D) might control blood pressure, but inhibition of the renin-angiotensin system is still the best answer. 9. Answer: D ‘A native arteriovenous fistula is the preferred access for chronic hemodialysis. If an arteriovenous fistula, ‘cannot be constructed, a synthetic arteriovenous graft (Answer C) is considered second line. A subclavian, catheter (Answer A) is a poor choice because of the increased risk of infection and thrombosis and because ‘of the poor blood flows obtained through a catheter. ‘A Tenckhoff catheter (Answer B) is incorrect because ‘this is a catheter for peritoneal dialysis © 2009 American College of Clinical Pharmacy 2.28 30. Answer: D ‘The most well-studied medication is midodtin agonist. Levocaritine (Answer A) has been tried, but there are limited data on its benefit. Fludrocortisone (Answer B) isa synthetic minerlocorticoid, which is used for hypotension in other situations; however, the primary mechanism is due to Na and water restriction inthe kidney; hence, this medication is less likely to work. Sodium chloride tablets (Answer B) would not ‘work acutely and should generally be avoided. 11. Answer: B Hyperparathyroidism is associated with epoetin resistance in hemodialysis patients (Answer B). Although iron deficiency isthe most common cause of epoetin deficiency the laboratory results in ths patient do not indicate iron deficiency (Answer A). Phenytoin therapy (Answer C) has been associated with anemia in other patient populations but not in hemodialysis patients. Infection (Answer D) and inflammation are very common causes of epoetin deficiency in patients ‘on hemodialysis, but there is nothing in this patient's presentation to suggest an infectious or inflammatory process. 12, Answer: B D.W. requires treatment for his elevated iPTH (800 pg! AL), which putshimathighrisk renal osteodystrophy. He has high serum phosphorous and calcium. The ‘corrected calcium is 10.2 mg/dL, and the calcium x phosphorus factor is 80 mg/dL?” The goalarget Ca % Phos factor in stage V CKD is less than 55 mg?! A ppatient with DM loses wages because of missed work related to her illness. Which one of the following is an example of the lost wages? Direct medical cost. Direct nonmedical cost. Indirect cost. Intangible cost. paR> © 2009 American College of Clinical Pharmacy Endocrine and Metabolic Disorders Ina recent study of aspirin use among adults with DM, the authors found an adjusted odds ratio of aspirin use of 4.3 (95% confidence interval [Cl] = 1.6-1.8) among patients with established cardiovascular disease and 1.4 (95% Cl=0.4-5.1) among patients with four to six cardiovascular risk factors without established cardiovascular disease. Which one of the following is the best interpretation of the data? A. Patients with multiple risk factors are significantly more likely to be taking aspirin compared with individuals without risk factors because the CI does not include zero. B. Aspirin significantly reduces the risk of cardiovascular events in patients with DM. ._ Patients with cardiovascular disease are more likely to take aspirin compared with patients without cardiovascular disease because the ‘Cl does not include 1. D. Patients taking aspirin are more likely to develop cardiovascular disease. KM. is a 42.yearold obese African American ‘woman with DM, hypertension, and depression. Her blood pressure (BP) measurements for the past three office visits areas follows: 156/92 mm. Hg, 174/110 mm Hig, and 168/108 mm Hg. Her i positive for trace protein. Her last A, was 11.3%, She does not follow a diet or exercise routinely. Drugs include 81 mg/day of aspirin, lisinopril-hydrochlorothiazide 20/25, metformin 2000 maiday, and glyburide 10 mg/day. Which ‘one of the following interventions is eritical to lower her risk of macrovascular disease, based on the UK. Prospective Diabetes Study trial? A. Tight glycemic control with an A,, of 6.5% or less. B. Any improvement in her glycemic control. C. Adding’ an angiotensin II receptor blocker (ARB) to decrease the protein in her urine. D. Lowerher BP. JS. is a 49-year-old African American obese woman with type 2 DM, hypertension, and dyslipidemia. Her current antihypertensive regimen includes hydrochlorothiazide 12.5 mg/ day. She did not tolerate angiotensin-converting enzyme (ACE) inhibitor or ARB therapy. Vital signs include BP 136/82 mm Hg with a large ccf sitting and pulse rate 72 beats‘minute. Her albumin-to-creatinine ratio was 55 meg/mg, and in a repeat test 2 months later, it was 102 meg/ ‘mg, Her serum creatinine was 1.0. Which one of the following therapeutic actions should she be ‘considered for, based on the American Diabetes Association (ADA) guidelines for diabetic nephropathy? ‘A. Add clonidine B. Add a nondihydropyridine calcium channel blocker. C. Add an a-blocker. D. Discontinue the diuretic. TK. is a 3S-yearold underweight woman presenting to the office for her general health ‘maintenance. She presently takes levothyroxine 0.2. mglday. Her thyroid-stimulating hormone (TSH) concentration returns to below normal. ‘Which one ofthe followingisthe bestmanagement step for this patient? ‘A. Repeat the TSH and continue the present levothyroxine dose, B. Order a serum thyroxine (T), repeat the ‘TSH, and continue the present levothyroxi dose. CC. Ordera serum T, decrease her levothyroxine dose, and recheck the TSH in 6 weeks. D._ Decrease her levothyroxine dose and recheck the TSH in 6 weeks. © 2009 American College of Clinical Pharmacy 2.33 Endocrine and Metabolic Disorders Note: Throughout this chapter, you will see the letters a, b, c, or e at the end of certain lines, which correspond to the “American Diabetes Association Standards of Medical Care, Levels of Evidence.” American Diabetes Association evidence grading system for clinical practice recommendations: a= Clear evidence; well-conducted, randomized controlled trials or meta-analysis. b= Supportive evidence; well-conducted cohort studies. ¢= Supportive evidence from poorly controlled or uncontrolled studies. = Expert consensus or clinical experience. I, DIABETES MELLITUS ‘A. Etiologic Classifications of DM 1. Type! a. Results from f-cell destruction b, Absolute lack of insulin 2. Type2 a. Results from progressive insulin secretory defect and insulin resistance b. Relative lack of insulin Gestational DM Onset or discovery of glucose intolerance during pregnancy Other specific types of diabetes a. Secondary causes i, Drugs and chemicals (a) Glucocorticoids: increase gluconeogenesis (b) Pentamidine: pancreatic toxicity (©) Protease inhibitors (@ Atypical antipsychotics Pancreatic disease iii. Cystic fibrosis ,. Endocrinopathies (a) Cushing’s syndrome (b) Hyperthyroidism vee B. Diagnosis of Nonpregnant Adults 1. Casual plasma glucose of 200 mg/dL or more (11.1 mmol/L) and symptoms, a. Casual is defined as any time of day without regard to the last meal b. Classic symptoms include polyuria, polydipsia, and unexplained weight loss. OR 2. Fasting plasma glucose (FPG) of 126 mg/dL or more (7.0 mmol/L) Preferred test to diagnose diabetes in children and nonpregnant adults (e) Fasting is defined as no caloric intake for at least 8 hours. Preferred test for nonpregnant adults and children Impaired fasting glucose: 100 mg/dL (5.6 mmol/L) to 125 mg/L (IFG and IGT are termed prediabetes) aoee OR 3. ‘Two-hour postglucose of 200 mg/dL or more (11.1 mmoV/L) during an oral glucose tolerance test a. Atotal of 75 g of anhydrous glucose dissolved in water © 2009 American College of Clinical Pharmacy 234 Endocrine and Metabolic Disorders . Impaired glucose tolerance: 2-hour results of 140-199 mg/dL c. Not recommended for routine use (less reproducible results) 4, Tests should be confirmed by repeat testing on a different day. a. Unless unequivocal symptoms of hyperglycemia are present b. Using A, for diagnosis is not recommended at this time. (e) C. Testing for Prediabetes and Diabetes Guidelines a. Type 1: only if symptomatic (usual presentation is more acute) b. Type 2: Screen high-risk individuals; if values are normal, retest in at least 3-year intervals. (e) Criteria for testing asymptomatic adult individuals to detect prediabetes and type 2 DM a. Testing should begin in all adults 45 years or older. (b) b. Consider testing at a younger age if overweight or obese (body mass index [BMI] of 25 kg/m? or more) with one or more additional risk factors for diabetes i, Habituaily physically inactive ii, A first-degree relative with DM iii, High-risk ethnic population (e.g., African American, Latin American, Native American, Asian American, Pacific Islander) iv. Delivered a baby weighing more than 9 Ib ot with gestational DM v. Hypertensive (140/90 mm Hg or more) vi. High-density lipoprotein of 35 mg/dL or less (0.90 mmol/L) and/or TG of 250 mg/dL. or more (2.82 mmol/L) Polycystic ovarian syndrome Impaired fasting glucose or IGT on previous test ix. History of vascular disease cc. Test for prediabetes or diabetes: an FPG test, a 2-hour oral glucose tolerance test (75-g glucose load), or both (b) i. Amoral glucose tolerance test may be considered in those with normal IFG to better define the diabetes risk. (¢) ii, In those with prediabetes, if appropriate, treat other cardiovascular risk factors. (b) iii, For individuals with prediabetes, retest for diabetes yearly. (e) Detection of gestational DM a. Risk factor assessment at the initial visit i, Very high-risk women (severe obesity, glycosuria, or strong family history of type 2 DM; polycystic ovarian syndrome; history of gestational DM; or delivery of a large- for-gestational-age infant) ii, Ifappropriate, complete an oral glucose tolerance test, (¢) b, Screen/diagnose between 24 and 28 weeks of gestation, c. Diagnosis with 100 g of oral glucose tolerance test (may use a two-step approach by screening with a 50-g glucose load) 4. Low-risk women are not required to be tested, but low-risk designations are limited to women meeting all of the following criteria: younger than 25 years, weight normal before pregnancy, member of an ethnic group with a low prevalence of diabetes, no first-degree relative with diabetes, and no history of abnormal glucose tolerance or poor obstetric outcome. © 2009 American College of Clinical Pharmacy 2.35 Endocrine and Metabolic Disorders 4, Prevention/delay of type 2 DM a. Lifestyle counseling b. Metformin may be considered in very high-risk individuals. i. Combined IFG and IGT with additional risk factors and are obese and younger than 60 years (e) Patient Case ‘A 48-year-old African American man with a family history of DM is seen in the office, and an FPG of 145 mg/ L is obtained. He denies any specific complaints but notes a 10-Ib weight gain during the past year, Which one of the following is the best approach to managing this patient at this time? A B. c. D. Diagnose DM and begin therapy. Repeat the FPG. Obtain an A,,. Rescreen in § years. D. Recommendations for Adults with Diabetes Glycemic control Key points of setting glycemic goals: Goals are individualized. In general, for nonpregnant adults, target an A,_of about 7% to lower the risk of microvascular and neuropathic complications in type I and type 2 DM. (a) iii. The general goal of less than 7% for many adults to lower the risk of mac:nvascular complications (b) iv. AnA,, closer to normal (less than 6%) without significant hypoglycemia may be considered for some individuals. (b) Examples may include those newly diagnosed, those with long life expectancy, and those without cardiovascular disease. ¥. Certain populations (¢.g., children, pregnant women, the elderly) require special considerations. vi, Less-intense goals may be needed for patients with a history of severe hypoglycemia, limited life expectancies, advanced microvascular or macrovascular complications, extensive comorbid conditions, and long-standing diabetes in whom the general goal is difficult to attain, (¢) vii, Postprandial glucose should be targeted in patients who meet fasting goals but in whom the A,, remains elevated. Table 1. ADA Guidelines Parameter Nondiabetie Goal AS 3a toa rs see] eee) Sy ve ES 1 q IV G@ 200 mit ae awar) [cam [one a vee) [Gem] [eres = trmreds gre =) [Eeeeee] caster =) Geer! Lana + Double Give To was Repeat NaHCO __ tH Fe] FESS emai 414 allkgh 14mg 7 7 Monitor K ; ; erm tamer 1 Tar TT | SE Figure 2. ‘BS =blood sugar (blood glucose); D5 = 5% dextrose; IV = intravenous; K = potassium; NaHCO, = sodium bicarbonate; ‘NS = normal saline; SQ = subcutaneously. Reprinted with permission from the American Diabetes Association. Clinical practice recommendations —2004. Diabetes Care 2004;27:596, Available at www.diabetes.org. © 2009 American College of Clinical Pharmacy 2-48 Endocrine and Metabolic Disorders D. E. Potassium (K) Supplementation 1. Average K deficit: 3-10 mEq/kg 2. May present with normal or high serum K because of acidosis 3. Potassium concentration will fall with therapy because of: a. Dilution b. Continued loss in urine ©. Correction of acidosis 4. Insulin-mediated cellular uptake 4, Add up to 20-30 mEq of K+ per liter of intravenous fluid depending on serum K, renal function, and rate of intravenous fluid administration; K administration should not exceed 10-20 mEq/hour. ‘Sodium Bicarbonate (NaHCO,) 1, Indicated for treatment of severe metabolic acidosis only (pH less than 7.0) 2. Acidosis will usually correct itself with adequate insulin administration. & A B. c. D. Patient Case 23-year-old woman with type 1 DM presents with complaints of nausea, vomit i, and progressive ‘weakness. She discontinued her regularly scheduled insulin regimen (NPH and regular insulin 2 timesiday) because of an inability to keep down food. Physical examination reveals orthostatic hypotension, respiratory rate 34 breaths/minute, pulse rate 120 beats/minute, and mild abdominal tenderness. Laboratory values: Na 128;K 5.2; CLI glucose, and protei ‘Which one of the following should be part of her initial therapy? A 0, 12; BUN 35; serum creatinine 1.8; and BG 525 mg/dL; urinalysis: positive for ketones, “Arteria lod gases on oom st pO, 98; pH 125 pC, 28 HCO, 1 and Oa 38 Reinitiate her home insulin regimen. ‘NaHCO, 50 mEq intravenously to treat the metabolic acidosis Fluid restriction to treat the hyponatremia, 0.9% NaCl intravenously at 350 mL/hout. IIL NONKETOTIC, HYPERGLYCEMIC HYPEROSMOLAR STATE, A Associated with Severe Hyperglycemia, Severe Volume Depletion, and Very High Serum Osmolality Calculate Serum Osmolality Based On: 1. Serum osmolarity = 2[Na+] + [BG]/18 + [blood urea nitrogen {BUN})/2.8 (normal: 280-320 mOsm/L) Fluid Replacement 1. Fluid deficit = 15%-20% body weight, average 8-12 L 2. May begin with 0.9% NaCl to replace intravascular volume, 500-1000 mL/hour 3. Switch to 0.45% NaCl when hemodynamically stable. Insulin Administration © 2009 American College of Clinical Pharmacy 2-49 Endocrine and Metabolic Disorders IV. PREVENTION AND MANAGEMENT OF DIABETES COMPLICATIONS A. Aspirin Administration: 1. Use aspirin as a secondary prevention strategy in men and women with DM who have evidence of lange vessel disease, including those with a history of myocardial infarction, vascular bypass procedure, stroke, transient ischemic attack, peripheral vascular disease, claudication, and/or angina. (a) i. Secondary prevention and documented aspirin allergy, clopidogrel 75 mg/day should be used (c) Use aspirin as a primary prevention strategy for those with type 1 or type 2 DM at increased cardiovascular risk, including patients who are 40 years and older or who have additional risk factors (family history of cardiovascular disease, hypertension, smoking, dyslipidemia, or albuminuria). (c) Use aspirin in doses of 75-162 mg/day. Aspirin therapy should not be used in patients with: a. True aspirin allergy, asthma with aspirin sensitivity, bleeding tendency, anticoagulation therapy, recent GI bleeding, clinically active hepatic disease (e) b. Anyone younger than 21 years because of increased risk of Reye’s syndrome ‘Combination aspirin 75-162 mg/day and clopidogrel 75 mg/day is reasonable for up to 1 year after acute coronary syndrome. (b) B. Smoking Cessation: Diabetes Care 2004;27(Suppl 1):S24 1. 2. ‘Advise all patients to stop smoking. (a) Include smoking cessation counseling and other forms of treatment as part of routine care. (b) C. Blood Pressure: 2. 3. ‘Measured routinely at each visit ‘a, Systolic BP of 130 mm Hg or more or diastolic BP of 80 mm Hg or more on two separate occasions confirms the diagnosis of hypertension. (c) 'b. Complete orthostatic BP to assess for autonomic neuropathy (c) Goals ‘a, Systolic BP less than 130 mm Hg (c); diastolic BP less than 80 mm Hig (b); this is consistent with the Joint National Committee on Prevention, Detection, Evaluation and ‘Treatment of High Blood Pressure (INC VII). JAMA 2003;289:2560-72 ‘Treatment a. Systolic BP of 130-139 mm Hg or diastolic BP of 80-89 mm Hg i. Lifestyle modification for a maximum of 3 months ii If goals are not met, then treatment with pharmacologic agents may be added to lifestyle therapy. (e) b. Systolic BP of 140 mm Hg or more or diastolic BP of 90 mm Hg or more i, Pharmacologic therapy should be given. (a) Regimens should include an ACE inhibitor or ARB. If targets are not achieved, then a thiazide diuretic should be added if the glomerular filtration rate is 30 mL/minute per 1.73 m* or more. If the glomerular filtration rate is less than 30 mL/minute per 1.73 m?, then add a loop diuretic. (c) iv. Angiotensin-converting enzyme inhibitors and ARBs are contraindicated in pregnancy. © 2009 American College of Clinical Pharmacy 2-50 Endocrine and Metabolic Disorders Hyperlipidemia 1. Screen annually for most patients. (e) 2, Nonpharmacologic Treatment a. Nutrition: reduce saturated fat, trans fat, and cholesterol; promote weight loss, exercise (a) . Triglyceride concentrations are decreased with improved glycemic control. 3, Statin therapy should be added to lifestyle therapy regardless of baseline lipid levels for patients with DM: a. With overt cardiovascular disease (a) b. Without cardiovascular disease but older than 40 years and with an additional risk factor for cardiovascular disease (a) 4, Lower-risk individuals are not included in the above group (e.g., without cardiovascular disease, younger than 40 years). a. Statins may be considered if LDL is more than 100 mg/dL or if individuals have multiple cardiovascular disease risk factors. (e) 5. Goals a. Without overt cardiovascular disease, the primary goal is an LDL less than 100 mg/dL, (2.6 mmol/L). (a) b. With overt cardiovascular disease, an LDL less than 70 mg/L (1.8 mmol/L) using a high-dose statin is an option. (b) i, Statins are contraindicated in pregnancy. (e) ¢. For those on drug therapy, if the above treatment goals are not met on maximal-tolerated statin therapy, an optional goal of a reduction in LDL by about 30% 40% from baseline is an alternative therapeutic goal. (a) 4. If targets are not reached on maximal-tolerated statin therapy, combination therapy may be considered, but efficacy and safety have not been evaluated in cardiovascular outcomes studies. (e) e. Lowering 7G to less than 150 mg/L and raising HDL to more than 40 mg/dL in men (HDL more than 50 mg/dL in women) are desirable. However, statin therapy targeting the LDL lowering is the preferred strategy. (c) i, Current National Cholesterol Education Program (NCEP) guidelines suggest that, in patients with TG of 200 mg/dL or more, the non-HDL cholesterol (total cholestersl minus HDL) should be used. The non-HDL goal is 130 mg/AL or less. ‘Note: Sections 6, 7, and 8 below are not included in the 2009 ADA recommendations but are from the 2007 guidelines. The 2009 guidelines stress the use of statins and provide a table of studies to support statin use. Diabetes Care 2009;32:S30. 6. Combination therapy requires care to minimize adverse events; however, it may be needed. a. Combination therapy (statin/fibrate) has not been adequately evaluated for patient outcomes or safety. 'b. Gemfibrozil should not be initiated as monotherapy in mixed hyperlipidemia (high TG and LDL). 7. High-density lipoprotein cholesterol raising Lifestyle interventions: smoking cessation and exercise b. Niacin or fibrate therapy improves HDL more than statins; consider it if HDL is less than 40 and LDL is between 100 and 129 mg/dL. . Niacin: improves LDL, HDL, and TG but, at higher doses, may increase BG; modest doses ‘of 750-2000 mg/day may improve the lipid profile and affect the BG to a lesser degree © 2009 American College of Clinical Pharmacy 2.51 Endocrine and Metabolic Disorders 8. Triglyceride lowering a. Lifestyle interventions: decrease alcohol consumption; promote nutrition, weight loss; increase exercise ’b. Fibrate (gemfibrozil or fenofibrate), nicotinic acid, or high-dose statin E, Metabolic Syndrome (NCEP Adult Treatment Panel III guidelines) 1. Identification (three or more of the following risk factors) a. Abdominal obesity: waist circumference i, Men: more than 102 em (40 in.) ‘Women: more than 88 cm (35 in.) b. Triglyceride concentration of 150 mg/dL or more c. High-density lipoprotein i, Men: less than 40 mg/dL, ii, Women: less than 50 mg/dL, s Blood pressure 130/85 mm Hg or more Fasting glucose 110 mm Hg or more 2. Secondary target of risk reduction a. Reduce obesity and physical inactivity. b. Treat associated nonlipid and lipid risk factors (aspirin if indicated, BP). F. Coronary Heart Disease and Treatment 1. Screening a. Asymptomatic individuals stratify according to 10-year risk and treat risk factors accordingly (b) ‘Treatment a. Known cardiovascular disease—ACE inhibitor (c), aspirin (a), and statin (a) therapy to lower the risk of cardiovascular events b. Prior myocardial infarction: add a B-blocker to reduce mortality (a) ¢. Older than 40 years and additional cardiovascular risk factors (hypertension, family history, dyslipidemia, microalbuminuria, cardiac autonomic neuropathy, or smoking); aspirin and statin therapy to reduce cardiovascular events (b) ‘Treated CHF: thiazolidinediones are contraindicated. (c) Metformin may be used in stable CHF if renal function is normal. Do not use in unstable or hospitalized patients. (c) V. DIABETIC NEPHROPATHY ‘A. General Recommendations L. To reduce the risk and/or slow the progression of nephropathy, optimize glucose control. (a) 2. To reduce the risk and/or slow the progression of nephropathy, optimize BP control. (a) B. Screening 1, Patients with type 1: screen after 5 years (e) 2. Patients with type 2: screen at diagnosis (e) 3, Measure serum creatinine at least annually in adults with DM. 4, Screen with albumin-to-creatinine ratio (random spot collection is preferred method). a, At least two positive of three measurements in a 3- to 6-month period are required for diagnosis. © 2009 American College of Clinical Pharmacy 2-52 Endocrine and Metabolic Disorders C. Treatment 2. In the treatment of both micro- and macroalbuminuria, either ACE inhibitors or ARBs should be used. (a) Patients with type 1 DM who have with albuminuria and hypertension: data support use of an ACE inhibitor (a) Patients with type 2 DM who have microalbuminuria and hypertension: data support an ACE inhibitor and ARBs to delay progression to macroalbuminuria (a) Patients with type 2 DM with serum creatinine more than 1.5 mg/dL, hypertension, and ‘macroalbuminuria: data support an ARB to delay nephropathy progression (a) If one class is not tolerated, switch to another class. (e) With the presence of nephropathy, institute protein restriction. (b) Consider a calcium channel blocker or B-blocker or diuretic to further lower the BP or as an alternative for patients unable to tolerate an ACE inhibitor or ARB. ‘Table 6. Definitions of Abnormalities in Albumin Excretion ‘Category Spot Collection (meg/mg creatinine) ‘Normal <30 Microalbuminuria 30-299 (Macro (clinical) albuminuria 300 American Diabetes Association. Standards of medical care in diabetes—2009, Diabetes Care 2009;32(Suppl 1):S34. Available «at wwonediabetes.org. VI. DIABETIC RETINOPATHY A. General Recommendations 1 2, 3. Optimize glucose control. (Level of Evidence — A) Optimize BP control. (Level of Evidence ~ A) Aspirin therapy does not prevent retinopathy o increase the risk of hemorrhage. (Level of Evidence — A) B. Screening 1. 2 3. ‘Type 1: within 3-5 years of diagnosis with a dilated examination (Level of Evidence ~ B) ‘Type 2: shortly after diagnosis with a dilated examination (Level of Evidence ~ B) Annual examinations (Level of Evidence ~ B) VII. FOOT CARE A General foot examination should include the Semmes-Weinstein monofilament, tuning fork, palpation, and visual inspection. (Level of Evidence ~ B) ‘Comprehensive foot examination annually (Level of Evidence ~ E) Inspect feet at each office visit. (Level of Evidence ~ E) © 2009 American College of Clinical Pharmacy 2.53 Endocrine and Metabolic Disorders VIII. IMMUNIZATION ‘A. Influenza vaccine annually to all patients with DM age 6 months and older (c) B. Pneumococcal polysaccharide vaccine 1. All patients with DM 2 years and older (c) 2. Revaccinate adults 65 years or older if the first pneumococcal vaccination was given before age 65 and administered more than 5 years earlier. Other indications to revaccinate include nephrotic syndrome, chronic renal disease, and other immunocompromised states, such as, patients posttransplantation. IX. THYROID DISEASE A. Screening 1, Women older than 50 years a. Thyroid-stimulating hormone: less than 0.3-0.4 mU/L or more than 10 mU/L and then check free thyroxine (FT,) b. General screening is not recommended for men or women younger than 50 years. 2. Patients with type 1 DM: screen for thyroid peroxidase and thyroglobulin antibodies at diagnosis a. Thyroid-stimulating hormone should be measured after metabolic control is established. If normal, recheck every 1-2 years. b. AnFT, is measured if the TSH is abnormal. B. Physiology 1. The thyroid gland secretes two active hormones: triiodothyronine (T,) and T,. 2. Triiodothyronine is 4 times more potent than T,. 3. Most T, is produced from the peripheral conversion of T, to T,, C. Laboratory Assessment 1. Serum thyroxine a. Measures free and bound b. Altered by a variety of disease states and drugs c. Half-life = 7 days d._ Reference range = 4.5-12.5 meg/dL 2, Sensitive TSH Sensitive initial screening Independent of T,-binding globulin changes Can be abnormal before the onset of signs and symptoms Increased in primary hypothyroidism Decreased in primary hyperthyroidism Reference range = 0.4-5.5 mIU/L 3, Free serum thyroxine a. Direct measurements of FT, are more reliable than total T, and T, concentrations. b. Used to evaluate free concentrations in euthyroid-sick patients 4, Triiodothyronine uptake ‘a, Direct measure of the number of binding sites available on T,-binding globulin pepege © 2009 American College of Clinical Pharmacy 2-54 Endocrine and Metabolic Disorders b. Increased in hypothyroidism and when T,-binding globulin is increased ¢. Decreased in hyperthyroidism and when T,-binding globulin is decreased 4d. Reference range = 22%-34% 5. Free T, index a. Accurate estimate of FT, except in states of very high or very low T, binding globulin b. Caleulated: T, x TU or T, x 1/T,U c. Reference range = 1.0-4.3 6. Drug interactions with thyroid replacement therapy: Patient Case 9. A 72-year-old woman is scen in the emergency department for dizziness, palpitations, dyspnea, and fatigue. ‘She has had similar symptoms for the past 4 months. Her vital signs include a BP of 172/70 mm He sitting and a pulse rate of 122 beats/mimute noted to be irregularly irregular. An electrocardiogram shows atrial fibrillation, Her dose of levothyroxine is 0.15 mg/day. This dose was increased from 0.1 mgiday begun 4 weeks ago. A TSH returns the following morning at 0.01 mU/L. Which one of the following is the best management of this patient’s long-term thyroid replacement? ‘A. Continue present levothyroxine dose. B._ Increase dose by 25-50 megiday. C. Decrease dose by 25-50 megiday. D. Change to a combined T, and T, product. ‘Table 7. Drug Mechanism Clinical Intervention Bile acid sequesiranis [Decreased thyroid hormone absorption — | Separate doses by 4-6 hours due to binding (Oral contraceptives’ | Estrogen increases thyroid-binding | Monitor function, may need to increase estrogens globulin concentrations, resulting in| thyroid dose lowered free thyroid hormone Note: If estrogens are decreased, may need to decrease the thyroid dose Warfarin Thcreases the metabolism of clotting [Monitor INR: As thyroid dose increases, factors warfarin dose may need tobe decreased ‘Amiodarone Blocks conversion of T, to, and has | Treat hypothyroidism with thyroid high iodine content; may cause hypo- or replacement hyperthyroidism Lithium Tnhibition of synthesis and release of | Treat wit thyroid replacement if necessary thyroid hormone INR = international normalized ratio; T, = triiodothyronine; T, = thyroxine, D. Hyperthyroidism 1. Clinical presentation 2. Hyperthyroidism a. Heat intolerance; weight loss despite increased appetite; increased sweating . Palpitations; high-output failure; edema; increased pulse rate and systolic pressure; wide pulse pressure; presence of systolic murmurs, tachyarrhythmias c. Nervousness, irritability, emotional liability; insomnia or shortened sleep cycles © 2009 American College of Clinical Pharmacy 2.55 Endocrine and Metabolic Disorders E, Causes Elevated radioactive iodine uptake: true hyperthyroidism a. Graves’ disease: more common 1. i, Thyroid-stimulating antibodies activate like TSH. ii, Laboratory values: T,, T,U (or FT,), TSH iii, If not pregnant, check a 24-hour radioactive iodine uptake to confirm iv, Exophthalmus may be present. F. Treatment of Hyperthyroidism Table 8. Method Drug Dove MOA Foxielty ‘Comments Thioemides [PT 100-200 mg every 6| Blocks thyroid | Rashes; Drug of choice 50mg tablet hours Maintenance | hormone synthesis; |agramulocytosis; jin thyroid storm, maximum: 1200 | inhibits peripheral [Gl disturbance; | pregnant andor mg/day conversion of, 107, | hepatitis iacating [Methimazale 10-GOmgiday | Blocks thyroid __|SameasPTU _| Prefered agent Tapazote) Maintenance: | hormone synthesis [secreted in breast _| expect in above S-and 10-mg tablets |(once-daly dosing milk teratogenic | cases) is benefit) No eross- sensitivity with PTU with rash Todines [LugoTs solution: 8 | CugoT's: Blocks release of [Hypersensitivity [Use 7-14 days before mgidrop /$-10 drops 3 times!| thyroid hormone _|reactions—rashes, | surgery SSKI: day; SSKI: lor? | fom gland rhinorrhea, parotid | Do NOT use before 50 mg/drop drops 3 times/day Jand submaxillary | RAI—ase 3-7 days. swelling after RAT ‘Adrenergic | Propranolol [20-160 mg/day [Adjunctive therapy o | Bradycardia AV antagonist | tablets1V decrease the action of block ‘Nadolol tablets the thyroid hormone (Coriconerids | Dexamethasone, [Decreases thyroid Useful in thyroiditis prednisone, action; decrease and thyroid storm methylpredaisolone immune response in Hydrocortisone Graves’ disease AV = atrioventricular; GI= gastrointestinal; IV = intravenously; MOA = mechanism of action; PTU = propylthiouracil; RAT= radioactive iodine; SSKI = saturated solution of potassium iodide; G. Special Considerations Pregnancy: a. Due to Graves’ disease: 0.2% pregnancies b. Spontaneous abortion and fetal loss in untreated women ‘Treatment: propylthiouracil 2. Thyroid storm Life-threatening medical emergency: severe thyrotoxicosis Precipitating factors: infection, trauma, surgery, radioactive iodine treatment, and withdrawal from thioamides c. b ©. ‘Treatment teiodothyronine; T,~ thyroxine, i, Suppression of thyroid hormone formation and secretion: propylthiouracil-preferred thioamide, administer first after propylthiouracil Antiadrenergic therapy iii, Corticosteroids iv, Treatment of associated complications © 2009 American College of Clinical Pharmacy 2-56 Endocrine and Metabolic H. Hypothyroidism 1. Clinical presentation a. Deficiency of thyroid hormone b. Multisystem 2. Causes a. Hashimoto's thyroiditis i, Immunologic disorder in genetically predisposed individuals Iatrogenic causes i, Radioactive iodine/surgery Drugs (a) Todine-containing drugs: amiodarone Subclinical hypothyroidism ._ Elevated serum TSH concentration and normal FT, and T, concentrations without overt symptoms Risk of hypothyroidism is greatest in patients with multiple risk factors (women, older than 60 years, TSH more than 10, positive antibodies, history of thyroid disorder). iii, Thyroid replacement is controversial and dependent on symptoms. 4. Myxedema coma i, Long-standing uncorrected hypothyroidism ii, Treatment: intravenous T, 300-500 mcg, glucocorticoid therapy with intravenous hydrocortisone 100 mg every 8 hours, supportive therapy 1. Special Populations 1. Pregnancy: low thyroid hormone increases the risk of stillbirths a, Levothyroxine requirements are increased in pregnancy. J. Hypothyroidism Treatment: 1. Levothyroxine replacement a. Levothyroxine is the preparation of choice for replacement therapy. . Thyroid-stimulating hormone should be rechecked at least 6 weeks after a dosage change. ¢. Dosage adjustments are in increments of 12.5-25 meg/day. 4, Normalize to the TSH. K. Thyroxine Suppression Therapy 1, Indicated for nodular goiter, thyroid cancer L. Adverse Effects 1. Osteoporosis: a. Excessive replacement can cause hyperthyroidism, osteoporosis, and cardiac toxicity. X. ADRENAL DISORDERS ‘A. Cushing’s Syndrome 1, Defined as excess cortisol in the plasma © 2009 American College of Clinical Pharmacy 2.57 Endocrine and Metabolic Disorders . Presentation a. Obesity (90%), hypertension (85%), facial plethora (84%), glucose intolerance (80%), menstrual dysfunction (76%), hirsutism (72%), striae (67%), myopathy (65%), muscular weakness (58%), osteoporosis (55%) b. May present with low back pain and compression fractures of the spine . Etiologies (N Engl J Med 1996;335:1206-12, figure 1) a. Cushing’s syndrome b, Ectopic corticotropin syndrome ¢. Ectopic corticotropin-releasing hormone syndrome , Adrenal Cushing’s syndrome . Diagnosis of Cushing’s syndrome a, Two or three 24-hour urine collections for measurement of cortisol and creatinine b. Low-dose dexamethasone suppression test ‘Treatment a. Dependent on the etiology Corticosteroid preparations Table 9. Corticosteroid Equivalent Dose | Mineralocorticoid Activity (ong) (Godium-retaining potency) | [Cortisone 25 2+ [Hydrocortisone 20 2 Prednisone 3 1+ ‘Methylprednisolone 4 0 [Dexamethasone 075 0 (long duration of action) |. Corticosteroid dosing regimens Physiologic: equal to about 5 mg/day of prednisone b. Low supraphysiologic dose: 0.1-0.25 mg/kg/day c. High supraphysiologic dose: 1-3 mg/kg/day d._ Very high supraphysiologic dose: 15-30 mg/kg/day .. Dosing during stressful events a, Hypothalamic-pituitary-adrenal-suppressed patients i. Fifty milligrams of intravenous hydrocortisone every 6 hours on the day of surgery; lower doses or oral doses may be used for less stress ii, The minimum dose, dose interval, and therapy to suppress the hypothalamic- pituitary-adrenal axis have not been defined; however, after long-term hypothalamic- Pituitary-adrenal suppression, these can last up to 1 year. ). Alternative day therapy a, Minimizes adverse effects i. Hypothalamic-pituitary-adrenal suppression ii, Development or worsening of hypertension iii, Other long-term effects (a) Does NOT decrease the risk of developing DM or osteoporosis (b) Every-other-day dosing may not be appropriate for active retinoic acid or ulcerative colitis. ‘© 2009 American College of Clinical Pharmacy 258 Endocrine and Metabolic Disorders 10. Adverse effects of systemic corticosteroids ‘a. Short-term use: high-dose: cerebral edema, DM, GI bleed, glaucoma, hypertension, hypokalemic alkalosis, mood disorders, proximal myopathy, and Na and water retention 'b. Long-term use: amenorrhea, cataracts, central obesity, growth failure, hypothalamic- pituitary-adrenal suppression, hyperlipidemia, hypertension, immunosuppression, mood disorders, muscle weakness, and osteoporosis 11. Osteoporosis a. Pathophysiology i. Most rapid bone loss occurs in the first year of therapy. b. Prevention Prednisone 5 mg/day for 3 months or longer; calcium; vitamin D; weight-bearing exercise; and a bisphosphonate |. Adrenal Insufficiency 1. Primary adrenal insufficiency a. Addison’s disease is a chronic loss of adrenal function, b. Aldosterone secretion is not preserved (resulting in increased K+). c. Mineralocorticoid replacement may be necessary to decrease serum K, increase serum Na, or help with postural hypotension, ‘Secondary adrenal insufficiency a, Often related to exogenous corticosteroid replacement (e.g, abrupt discontinuation of long-term therapy) b. Aldosterone secretion is often maintained (because itis affected by the renin-angiotensin- aldosterone system). Presentation a. Primary and secondary adrenal insufficiency i, Tiredness, weakness, mental depression ii, Anorexia, weight loss iii, Dizziness, orthostasis, diarrhea iv. Hyponatremia, hypoglycemia, mild normocytic anemia ‘. Hyperpigmentation (primary insufficiency) Adrenal crisis a. Hydrocortisone 100 mg intravenously immediately and every 6-8 hours for 24-48 hours; then when stable b. Hydrocortisone 50 mg orally every 8 hours for 48 hours; taper to 30-50 mg/day . Intravenous normal saline supplementation to maintain hemodynamic support Glucocorticoid replacement dosing: two-thirds morning, one-third evening a. Hydrocortisone (drug of choice for Addison’s) 15-25 mg/day 'b. Prednisone about 2-3 mg in the morning and 12 mg in the evening ©. Fludrocortisone acetate: 0.05-0.2 mg added to decrease K+, to increase serum Na, or if postural hypotension is present Prevention a. Minimize systemic steroid use. b. Suppression of the hypothalamic-pituitary-adrenal axis occurs when supraphysiologic doses are taken, c. Abrupt discontinuation of steroids is usually safe when administered for less than 7-10 days. ‘© 2009 American College of Clinical Pharmacy 2.59 Endocrine and Metabolic Disorders REFERENCES Diabetes Mellitus 1 10, IL American Diabetes Association, Standards of ‘medical care in diabetes—2009. Diabetes Care 2009;32(Suppl 1):S13-S61. Available at www. diabetes.org. Nathan DM, Buse JB, Davidson MB, et al Medical management of hyperglycemia in type 2 diabetes: a consensus algorithm for the initiation and adjustment of therapy. Diabetes Care 2009;32:193-203. Available at www.diaberes. org. ‘The American Association of Clinical Endocrinologists. Medical guidelines for the management of diabetes mellitus: the AACE system for intensive diabetes self-management: 2002 update. Endocr Pract 200238 (Suppl 1):40— 82. Available at www.aace.com. ‘The Diabetes Control and Complications Trial Research Group. The effect of intensive treatment of diabetes on the development and progression of long-term complications in insulin-dependent diabetes mellitus. N Engl J Med 1993;329:977- 86. UK. Prospective Diabetes Study Group. Intensive blood-glucose control with sulfonylureas or insulin compared with conventional treatment and risk of complications in type 2 diabetes (UKPDS 33). Lancet 1998;352:837-53. UK. Prospective Diabetes Study Group: Effect of intensive blood-glucose control with metformin ‘on patients with type 2 diabetes (UKPDS 34). Lancet 1998,352:854-65. ‘Tiplitt CL, Reasner CA, Isley WL. Diabetes mellitus. In: DiPiro JT, ed. Pharmacotherapy: A Pathophysiologic Approach, 7th ed. Stamford, CT: McGraw-Hill, 2008:1205—41, American Diabetes Association. Diabetic nephropathy. Diabetes Care 2004;27(Suppl 1)3879-S83, American Diabetes Association. Diabetic neuropathy. Diabetes Care 2004;27(Suppl IS. American Diabetes Association. Aspirin therapy. Diabetes Care 2004;27(Suppl 1):S72-S73, American Diabetes Association, Dyslipidemia management in adults with diabetes. Diabetes ‘Care 2004:27(Suppl 1):S68-S71 2. 2B. 14 15. 16. 1. 18. 19. 20. Bakris GL, Williams M, Dworkin L, et al Preserving’ renal function in adults with hypertension and diabetes: a consensus approach. National Kidney Foundation Hypertension and Diabetes Executive Committees Working Group. ‘Am J Kidney Dis 2000;36:646-61. Dewitt DE, Hirsch IB. Outpatient insulin therapy in type 1 and type 2 diabetes mellitus: scientific review. JAMA 2003;289:2254-64, Dewitt DE, Dugdale DC. Using insulin strategies in the outpatient treatment of diabetes: clinical applications. JAMA 2003;289:2265-9. Hirsch IB. Insulin analogues. 2005;352:174-83, Mooradian AD, Bernbaum M, Albert SG. Narrative review: a rational approach to starting insulin therapy. Ann Intern Med 2006;145:125- 34. Nathan DM, Buse JB, Davidson MB, et al. ‘American Diabetes Association. Management of hyperglycemia in type 2 diabetes: a consensus algorithm for the initiation and adjustment of therapy. Diabetes Care 2008:31(Suppl 1):173-<. US FoodandDrug Administration Rosiglitazone maleate (marked as Avandia, Avandamet, and Avandary)) Available at www.fda gow/ederidrug/ infopage/rosiglitazone/default.htm. Accessed February 5, 2009, Choy CK, Rodgers JE, Nappi JM, et al ‘Type 2 diabetes mellitus and heart’ failure. Pharmacotherapy 2008;28:170-92. Goldberg RB, Holman R, Drucker DJ. Management of type 2 diabetes. N Engl J Med 2008;358:293-7, JAMA ‘Thyroid Disease 1 Talbert RL. Thyroid disorders. In: DiPiro JT, ed. Pharmacotherapy: A Pathophysiologic Approach, ‘7th ed. Stamford, CT: MeGraw-Hill, 2008:1243— 6. Screening and management of thyroid disease. ‘Am Fam Physician June 15, 2000, Available at ‘www.aafp.org/afp/20000215/1047 hemi. Accessed February 5, 2009. US, Preventive Services Task Force. Screening for thyroid disease: recommendation statement. ‘Ann Intern Med 2004:140:125-7. Available at www.ahrg.govelinic/rduspstfithyroid/thyrrs. sum, Accessed February 5, 2009. © 2009 American College of Clinical Pharmacy 2-60 Endocrine and Metabolic Disorders 4, Surks MI, Ortiz E, Daniels GH, etal. Subclinical thyroid disease: scientific review and guidelines for diagnosis and management. JAMA. 2004;291:228-38. Adrenal Insufficiency 1. Gums JG, Tovar JM. Adrenal gland disorders. In; DiPiro JT, ed. Pharmacotherapy: A Pathophysiologic Approach, 7th ed. Stamford, (CT: McGraw-Hill, 2008:1265-80, 2. Oclkers W. Adrenal insufficiency. N Engl J Med 1996;335:1206-12. 3. Krasner AS, Glucocorticoid-induced adrenal insufficiency. JAMA 1999;282:671-6, 4. Orth DN. Cushing's syndrome. N Engl J Med 1995;332:791-803, 5. American College of Rheumatology Task Force ‘on Osteoporosis Guidelines. Recommendations for the prevention and treatment of glucocorticoid- induced osteoporosis, Arthritis Rheum 2001;44:1496-503. © 2009 American College of Clinical Pharmacy 2-61 Endocrine and Metabolic Disorders ANSWERS AND EXPLANATIONS TO PATIENT CASES, 1. Answer: B ‘When screening patients for diabetes, if the patient's FPG exceeds 125 mg/dL, a diagnosis of DM should bbe suspected, but a repeat FPG should be obtained for confirmation before treatment is initiated. Although ‘other methods for screening include the oral glucose tolerance test, a fasting whole BG concentration, and ‘random BG concentration the FPG concentration is the preferred method. Testing of whole BG is primarily used for home monitoring and is not recommended for establishing the diagnosis of diabetes. The A,, is useful in monitoring but is not recommended for Screening or diagnosis. Ifa patient does not fulfill the eriteria for diabetes or IFG during screening, then a repeat screening is recommended every 3 years because of the low likelihood of developing diabetes and any of its complications in an individual with a negative screening result. American Diabetes Association. Standards of medical care in diabetes. Diabetes Care 2008;31(Suppl 1);S12-SI5, 2. Answer: A. Screening for diabetes is indicated for all adults who are overweight (BMI more than 25 kg/m’) and have additional risk factors. Fasting plasma glucose readings between 100 and 125 mg/dl. are defined as IFG. Intervention for those with IFG (¢) or IGT (a) includes counseling on weight loss and increased physical activity. Drug therapy, specifically metformin, may be considered to prevent the development of type 2 DM in those at very high risk: individuals with both TFG and IGT, obesity, additional risk factors, and younger than 60 years (Level of Evidence — E). Close attention should be given to other cardiovascular risk factors including tobacco abuse, hypertension, and hyperlipidemia. Monitoring for the development of diabetes should be performed every year in those with prediabetes. Diabetes Care 2008;31(Supp! 1):S15, 3. Answer: C ‘An SMBG should be used in conjunction with A, to ‘monitor petient goals. The goal for A,, is less than 79%, ‘and ideally, 2-hour postreadings should be less than 140 mg/dL. For patients with postprandial elevations, nutritional counseling may be beneficial. The addition of medication that helps lower postprandial BG (eg, acarbose) may be considered. Acarbose should be titrated slowly to avoid GI adverse effects. Liver function monitoring is required at higher doses. Metformin is contraindicated in patients with CHF treated with therapy and in men with a serum creatinine concentration of 1.5 mg/dL. or more. Because both repaglinide and glipizide bind to the same sulfonylurea receptor, this combination is not likely to provide additional benefit. 4, Answer: B First-line agents for the treatment of hyperglycemia may beselected based on efficacy, toxicity, and specific patient characteristics. Obese ‘patients should be initiated on metformin and titrated to an effective dose of about 2000 mg/day based on the UK. Prospective Diabetes Study 34 andsafetyprofile Inelderiy patients, the renal clearance of metformin is problematic and potentially places them at risk of developing lactic acidosis. Lean and elderly patients may respond better to an insulin secretagogue because these patients are ‘more likely to have insufficient secretion of insulin ‘than insulin resistance. Short-acting secretogogues ‘may have a lower risk of hypoglycemia than a long- acting sulfonylurea. Glyburide is more likely to cause hypoglycemia in renally impaired patients and the ‘elderly. Insulin is typically not considered a first-line ‘therapy in type 2 DM unless the patient presents with severe hyperglycemia. Oki JC, Isley WL. Diabetes mellitus. In: DiPiro JT, ed. Pharmacotherapy: A Pathophysiologic Approach, Sth ed. Stamford, CT: McGraw-Hill, 2002:1350, figure 74-6. S. Answer: B Hepatoxicity is a serious adverse effect of concern with the use of the thiazolidinediones. Troglitazone was removed from the market because of the risk of hepatotoxicity. The recommendations for liver function test monitoring have been changed from ALT at baseline and every 2 months for the first year to baseline and periodically as directed by clinical judgment. References: Actos package insert. Avandia package insert. © 2009 American College of Clinical Pharmacy 2-62 Endocrine and Metabolic Disorders 6. Answer: D For patients with type 2 DM, the addition of intermediate- or long-acting insulin is considered ‘when oral therapy has failed to achieve the glycemic goals. Regular or lispro insulin is used to cover meals in a typical insulin regimen and, therefore, is not appropriate at bedtime. The combination of rosiglitazone and insulin has been associated with the development of significant edema and potentially the development of CHF. The use of this combination is not indicated. Pioglitazone is US. Food and Drug Administration approved in combination with insulin; however, edema is also an adverse effect of pioglitazone. If patients are placed on combined slitazone and insulin therapy, they should be monitored for edema and the development of heart failure symptoms. Diabetes Care 2001;24:1226-32. Diabetes 2003;26(Suppl 1):S42. Available at www fda. ‘govleder/warn/warn2001 him. Accessed February 28, 2009. 7. Answer: D Intensive insulin regimens for patients with type 1 diabetes must allow flexibility of changes made in their dietary intake at each meal. A split-mix regimen ‘0f 70/30 insulin is too rigid and does not accommodate decreasing or omitting meals, Lowering the dose ‘of 70/30 insulin in the morning may help decrease midmorning hypoglycemia, but this may result in loss of control in the afternoon and before-dinner zlucose. For a patient with type 1 DM, use a regimen that provides basal coverage with long-acti (neutral protamine Hagedorn [NPH] insulin or insulin, ‘glargine) combined with short-acting insulin (lispro Or aspart) before meals. The short-acting insulin can, be varied based on carbohydrate intake. 8. Answer: Initial treatment of diabetic ketoacidosis includes aggressive fluid replacement, high-dose insulin ‘administration, and K supplementation as needed. Fluid supplementation is critical to stabilize the patient's hemodynamic status and begin to lower BG concentrations. Insulin can be administered subcutaneously, but more often, intravenous doses are given for a more consistent effect that can be titrated. Serum Na should be corrected for hyperglycemia (or cach 100 mg/dL of glucose more then 100 mg/ AL, add 16 mEq to the Na value for the correct serum Na value). Sodium bicarbonate is reserved for patients with severe metabolic acidosis, pH less than 70, American Diabetes Association. Hyperglycemic crises in diabetes. Diabetes Care 2004;27(Suppl 1)S94-S102, 9. Answer: C Excessive thyroid replacement can result in symptoms of hyperthyroidism (eg, bone mineral density loss, osteoporosis, palpitations, high output failure, increased pulse rate, a wide pulse pressure, tachyarthythmias). With the development of sensitive TSH, levothyroxine replacement can be accurately ‘monitored. Suppression of the TSH indicates ‘excessive replacement. The average maintenance dose is 110-120 mogiday. As individuals age, thyroid hhormone clearance declines, and the requirements for thyroid replacement may decline. Elderly patients may be more sensitive to the cardiac effects and require lower initial doses (¢g., 25 megiday). Dosage changes are typically made in increments of 12.5-25 ‘megiday. Talbert RL. Thyroid disorders. In: DiPiro JT, ced. Pharmacotherapy: A Pathophysiologic Approach, 6th ed. Stamford, CT: McGraw-Hill, 2005:1368. ‘© 2009 American College of Clinical Pharmacy 2-63 Endocrine and Metabolic Disorders ANSWERS AND EXPLANATIONS TO SELF-ASSESSMENT QUESTIONS 1. Answer: D ‘The diagnosis of DM is made in one of three ways: (1) two fasting plasma readings of 126 mg/dL or ‘more; (2) a casual reading of 200 mg/dL or more with symptoms; or (3) a 2-hour post-oral glucose tolerance test reading of 200 mg/dL or more. Patients ‘with multiple risk factors such as obesity, African ‘American rece, older than 45 years, bypertensica, and dyslipidemia (elevated TG and low HDL) should ideally be screened with an FPG. Diabetes Care 2008;31(Suppl 1yS12-SI5. 2. Answer: D Metabolic syndrome is recognized as a group of ‘major cardiovascular risk factors. It is possible that the syndrome is closely related to insulin resistance. ‘The characteristics identifying the disorder include (hypertension, (2) abdominal obesity, and G) dyslipidemia (levated TG and low HDL) as well as fasting glucose of 110 mg/dL. or more. Three or more characteristics are needed. The target of therapy is to decrease insulin resistance by increasing physical activity and weight loss. Third Report of the NCEP Expert Panel on Detection, Evaluation, and Treatment cof High Cholesterol in Adults (Adult Treatment Panel uD. 3. Answer: C ‘An SMBG is important in the assessment and management of glycemic control. The A, should correlate with the SMBG. For each 30 mg/dL more than 120 mpidL, the A,,is about 1%. For example, an A,, of 6% correlates t8 mean whole blood concentration’ of 120 mgldL. An A, of 109% would be expected to correlate with SMBO concentrations of 270 mg/dL. Discrepancies between the SMBG readings can occur ifthe meters not working properly. The A, may also be elevated if there was a recent ilinss thet has since resolved, Diabetes Care 2008:31(Supp! DSI. 4. Answer: A The development of type 1 DM most often occurs in individuals younger than 30 years. The presentation in older adults may be more insidious because the autoimmune process slowly destroys the pancreatic B.islet cells. Once a critical mass of insulin-producing cells is lst (about 90%), normoglycemiais no longer maintained. The presence of Ketones in the urine suggests a lack of insulin. Appropriate therapy for the patient with type 1 DM is insulin therapy for survival. 5. Answer: C Direct medical costs refer to the resources used for detecting, treating, or preventing a medical condition. Direct nonmedical costs include the cost for nonmedical services or items, such as transportation, food, and family care, which are incurred because of theillness. Indirect costs are due to loss of productivity because of an illness or premature death. Intangible ‘costs are those that cannot be directly measured in monetary terms but occur because of the illness (e.g. pain and suffering). Carter BL, Lake K, Raebel M, et al, eds. Pharmacotherapy Self-Assessment Program, 3rd ed. Research, Biostatistics, and Drug Information ‘Applications Module. Kansas City, MO: ACCP 1999:73-102, 6. Answer: C In this study, the odds ratio is used to describe the strength of an association between the presence of cardiovascular risk factors or disease and aspirin use. ‘An odds ratio with Cls that include 1.0 indicetes that patients are no more likely to be taking aspirin if they have the multiple risk factors than if they do not. In this case, the study is not evaluating the efficacy of aspirin and the odds (or risk) of cardiovascular disease in patients taking aspirin, but rather the use of aspirin in patients with cardiovascular disease. References: Diabetes Care 2001;24:197-201. Hayney MS, Meck PD. Essential clinical concepts of biostatistics. In: Carter BL, Lake K, Raebel M, et al, eds. Pharmacotherapy Self Assessment Program, 3rd ed. Research, Biostatistics, ‘and Drug Information Applications Module. Kansas City, MO: ACCP, 1999:19-42. 7, Answer: D ‘The UK. Prospective Diabetes Study results demonstrated that the risk of microvascular complications (retinopathy, nephropathy, and possibly neuropathy) is lessened by lowering BG Concentrations in patients with type 2 DM. with intensive therapy. No threshold was identified for the ‘benefits of tight control for these outcomes. These results confirm findings in the Diabetes Control and ‘Complications Trial for patients with type 1 DM. In the UK. Prospective Diabetes Study results, a 16% reduction in cardiovascular complications did not reach statistical significance. The study showed that lowering BP to a mean of 144/82 mm Hg significantly reduced strokes, DM-related deaths, and heart failure. ‘Although microalbuminuria has’ been correlated ‘with an increased risk of cardiovascular disease, the addition of an ARB to lower protein inthe urine has notbeen shown to decrease thepatient’s cardiovascular risk, Diabetes Care 2003:26:S28-S32. © 2009 American College of Clinical Pharmacy 2-64 Endocrine and Metabolic Disorders 8. Answer: B The most recent ADA guidelines suggest that the albumin-to-ereatinine ratio in a random spot collection is the preferred method to screen for ‘microalbuminuria. If the rato is more than 30 but less than 300 on at least two of three measures within 3-6 ‘months, the diagnosis can be made. Based on the most recent ADA guidelines, initial drug therapy to delay the progression from micro- to macroalbuminuria should begin with an ACE inhibitor or an ARB in the patient with type 2 DM (with or without_ hypertension). Second-line therapy, if an initial ACE inhibitor or ARB therapy is not tolerated, is a calcium channel blocker, diuretic, or B-blocker to lower BP. Blood slucose control and BP control should be optimized. Diabetes Care 2008;31(Suppl 1)S29-S30. 9. Answer: D In the management of hypothyroidism, the goal is to normalize the TSH. Overreplacement of T, can lead to bone loss and potentially osteoporosis. A suppressed TSH can identify excessive replacement. Underreplacement may be detected with an elevated TSH. Changes in levothyroxine replacement should ‘be made in increments of 12.525 meg. Because of the very long halflife of levothyroxine, the TSH is rechecked at least 6 weeks after a dosage adjustment. Suppression of the TSH with excessive levothyroxine replacement may be used in patients with nodular thyroid disease, diffuse goiter, a history of thyroid irradiation, or thyroid cancer. Serum T, concentrations can be useful when nonadherence ig suspected, but these concentrations are not routinely needed to monitor levothyroxine therapy. Talbert RL. ‘Thyroid disorders. In: DiPiro JT, ed. Pharmacotherapy: ‘A Pathophysiologic Approach, 6th ed. Stamford, CT: “McGraw-Hill, 2005:1369, © 2009 American College of Clinical Pharmacy 2.65 Endocrine and Metabolic Disorders NOTES © 2009 American College of Clinical Pharmacy 2-66 FLUIDS, ELECTROLYTES, AND NUTRITION JUDITH L. KRISTELLER, PHARM.D., BCPS WILKES UNIVERSITY WILKES-BARRE, PENNSYLVANIA GORDAN S. SACKS, PHARM.D., BCNSP, FCCP UNIVERSITY OF WISCONSIN - MADISON MADISON, WISCONSIN FLUIDS, ELECTROLYTES, AND NUTRITION JUDITH L. KRISTELLER, PHARM.D., BCPS WILKES UNIVERSITY WILKES-BARRE, PENNSYLVANIA GORDAN S. SACKS, PHARM.D., BCNSP, FCCP UNIVERSITY OF WISCONSIN - MADISON MADISON, WISCONSIN © 2009 American College of Clinical Pharmacy 2-67 Fluids, Electrolytes, and Nutrition Learning Objectives: 1. Calculate the osmolarity of intravascular fluids ‘and compare with normal plasma osmolarity. 2. Recommend an appropriate intravenous fluid regimen and monitoring parameters based on a patient's clinical characteristics. 3. Discuss theappropriateuse andrisks ofhypertoni and hypotonic saline, and recommend a treatment regimen and monitoring parameters to ensure safe and effective use of these intravascular fluids. 4. Assess electrolyte abnormalities and recommend ‘an appropriate pharmacologic treatment plan based on individual patient signs and symptoms. 5. Discuss appropriate indications for the use of enteral and parenteral nutrition, 6 Recommend a patient-specific enteral formula, infusion rate, and monitoring parameters. 7. Recommendapatient-specific parenteral nutrition formula and monitoring plan based on type of vascular access, nutritional needs, comorbidities, and clinical condition. 8. Discuss strategies for preventing compli associated with enteral and parenteral nutrition. Self-Assessment Questions: Answers to these questions may be found at the end of this chapter 1, A 74-year-old woman presents to the emergency department with a 3-day history of cough, temperature of 102°F, and lethargy. She has the following vital signs and laboratory values: blood pressure (BP) 72/40 mm Hg, heart rate 115 beats/ minute, white blood cell (Count) (WBC) 18,000, blood urea nitrogen (BUN)/creatinine (Cr) 28/17 ‘mgidL (baseline Cr 1.2 mg/dL), weight 72 kg. After a 500 mL fluid bolus of 0.9% sodium chloride (NaC), her BP increased to 82/48 mm Hg. Her chest radiography is consistent with ‘pneumonia. Her history includes coronary artery disease and arthritis. Which one of the following. is the best initial intravenous treatment strategy? ‘A. 500 mL fiuid bolus with albumin 5%. B. 1000 mL fluid bolus with 5% dextrose (D,W) 0.45% NaCl C. 019% NaC! to infuse at 100 mL/hour. D. 1000 mL fluid bolus with 0.9% NaCl. ‘An order has been received for 3% NaCl. Using (0.9% NaCl and 23.4% NaCl, frst determine how such of each is needed to prepare | L of 3% NaCl. Second, calculate the osmolarity of 3% NaCl. Finally, determine if the resultant solution should bbe administered through a central or peripheral intravenous infusion. (Molecular weight [MW] ‘of NaCl is $8.5, and osmotic coefficient is 0.93.) A. Mix 907 mL of 0.9% NaCl with 93 mL of 234% NaCl; osmolarity = 954 mOsm/L; central intravenous infusion. B. Mix 907 mL of 0.9% NaCl with 93 mL of 23.4% NaCl; osmolarity = 1026 mOsm/L; central intravenous infusion C. Mix 850 mL of 0.9% NaCl with 150 mL of 23.4% NaCl; osmolarity = 954 mOsm/L; central intravenous infusion. D. Mix 850 mL of 0.9% NaCl with 150 mL of 23.4% NaCl, osmolarity = 1026 mOsm/L; peripheral intravenous infusion. ‘A 68-year-old man is admitted to the hospital for worsening shortness of breath during the past 2 weeks caused by heart failure, His serum Na concentration on admission was 123 mEq/L. Other abnormal laboratory values include brain natriuretic peptide 850 pg/mL and Cr 1.7 mg/dL. Chest radiography is consistent with pulmonary edema. The patient weighs 85 kg on admission, which is 3 kg more than his baseline weight. The patient is not experiencing nausea, headache, or mental status changes. The physician orders 3% NaCl intravenously to treat the hyponatremia, Which one of the following is best? A. 3% NaCl is an appropriate choice because the hyponatremia is likely acute. B. A 250-mL bolus of 3% NaCl is appropriate if used in combination with furosemide to avoid volume overload. CC. 3% NaCl is appropriate as long as the serum ‘Na does not rise more than 10 mEq/L in 24 hours. D. The risks of 3% NaCl outweigh the potential benefit for this patient. © 2009 American College of Clinical Pharmacy 2-68 Fluids, Electrolytes, and Nutrition 4. A 55-year-old man with diabetes mellitus and kidney disease presents with hyperkalemia. His laboratory values include potassium (K+) 72 mEqIL, calcium 9 mg/dL, albumin 3.5 g/L, and blood glucose 302 mg/dL. His electrocardiogram (EKG) is abnormal with peaked T waves Which ofthe following is the most appropriate recommendation for initial treatment of his hyperkalemia? ‘A. Regular insulin 10 units intravenously plus 50 g of glucose intravenously. B. 10% calcium gluconate 10 mL intravenously cover 5 minutes. C. Kayexalate 20 g mixed with 100 mL of 20% D. sorbitol orally every 4 hours as needed. Sodium bicarbonate 50 mEq intravenously over $ minutes. 5. A 68-year-old patient is admitted to the hospital after ‘@ cardioembolic. stroke. Her history is significant for atrial fibrillation, acute myocardial infarction, and diabetes mellitus. She has been unconscious for 48 hours. The medical team decides to start feeding the patient. Her weight is 60 kg, and all laboratory values including glucose concentrations are normal. She currently has no enteral access, but she does have # peripheral intravenous catheter. Which one of the following nutritional regimens is best for this patient? A. Initiate parenteral nutrition (PN) containing 60 g of amino acids (AAs), 500 mL of 10% lipid emulsion, 300 g of dextrose, standard electrolytes, ‘multivitamins, and trace clements in a total volume of 2000 mL administered over 24 hours. B. Initiate PN containing 40 g of AAs, 500 mL of 10% lipid emulsion, 200 g of dextrose, standard electrolytes, ‘multivitamins, and trace elements in a total volume of 2000 mL, administered over 24 hours. C. Insert a nasogastric (NG) or nasoduodenal feeding tube and infuse Isocal (1 keal/mL) starting at 25 mL/hour, advancing to a goal rate of 65 mL/hour. D. Insert a percutaneous endoscopic {gastrostomy feeding tube and infuse Isocal (1 cal/mL) starting at 25 mL/hour, advancing to.a goal rate of 100 mL/hour. ‘A 70-year-old man is admitted tothe hospital with peritonitis caused by severe inflammatory bowel disease. The patient has received adequate fuid resuscitation, and appropriate antibiotics have been prescribed. The physician, who wants the patient to undergo several days of bowel rest, has consulted with the pharmacist, who recommends a PN formula to be administered through a central line. The patient is. hemodynamically stable, with normal electrolyte concentrations Weight is 55 kg, prealbumin 20 mg/dL, BUN’ Cr 20/1. mg/dL, and WBC 17,000. Assuming appropriate electrolytes, multivitamins, and trace clements are included, which one ofthe following PN formulas when administered over a 24-hour period will provide this patient adequate calories, ‘AAS, and lipid? A. AA 10% 700 mL, dextrose 30% 325 mL, lipid 20% 500 mt. B. AA 10% 450 mL, dextrose 70% 400 mL, lipid 10% 500 mb. C. AA 10% 800 mL, dextrose 70% 350 mL, D. lipid 10% 500 mL. AA 15% 900 mL, dextrose 50% 500 mL, lipid 10% 500 mL. ‘A 40-yearcold man has been admitted to the hospital after several days of vomiting and iarthea. In the emergency department, he had several runs of nonsustained ventricular tachycardia. His plasma K+ on admission was 2.8 mEq/L. After receiving 200 mEq of KCI infused cover 24 hours, his repeat K+ is 32 mEq/L, and he continues to have runs of ventricular tachycardia. Other laboratory values include Na 143 mEq/L, magnesium 1.1 mEq/L, phosphorus 3 mg/ L, caleium 9 mg/L, and ionized calcium 1.1 mmol/L. Which one of the following suggestions is best to treat the hypokalemia? A. Administer KC! 20 mEq intravenously over 1 hour each x four doses and recheck Ki. B. Administer magnesium sulfate 2 g slow intravenous push. C. Administer K+ phosphate 15 mmol intravenously over 4 hours. D. Administer calcium gluconate 2g intravenously over 5 minutes. Which of the following nutritional strategies can prevent gut mucosal atrophy and subsequent bacterial translocation? PN enriched with glutamine, PN enriched with branched-chain AAs. Enteral nutrition (EN). Zine supplementation. pomp © 2009 American College of Clinical Pharmacy 2-69

You might also like